Schwarze Löcher gehören zu den astronomischen Objekten, an denen die Öffentlichkeit sehr große Interesse zeigt. Sie eignen sich wunderbar zum Spekulieren, sie haben ihren fixen Platz in Science-Fiction Geschichten, regen die Fantasie der Menschen an und machen ihnen auch ein bisschen Angst. Leider – oder vielleicht gerade deswegen – sind die allgemeinen Vorstellungen über schwarze Löcher durchsetzt von Mißverständnissen.

Am weitesten verbreitet ist die Vorstellung, dass schwarze Löcher einfach gnadenlos alles aufsaugen, was so im All rumschwirrt. Schwarze Löcher sind aber keine Staubsauger – sie üben wie jedes andere Objekt im Universum zwar auch eine anziehende Gravitationskraft aus, aber sie “saugen” nicht. Würde man die Sonne durch ein gleich schweres schwarzes Loch ersetzen, dann würde sich für die Erde genau nichts ändern (abgesehen davon dass es plötzlich dunkel und kalt würde). Sie würde weiterhin ihre Runden drehen; diesmal eben um das schwarze Loch. Die besonderen physikalischen Effekte des schwarzen Lochs entstehen durch dessen extreme Dichte und man bemerkt erst etwas davon, wenn man sich dem sogenannten “Ereignishorizont” nähert. Das ist die Grenze hinter der nichts mehr der Anziehungskraft des schwarzen Lochs entkommen kann. Egal wie schnell man sich bewegt – selbst wenn es mit Lichtgeschwindigkeit ist – man wird die Fluchtgeschwindigkeit nicht mehr erreichen und ist im schwarzen Loch gefangen.

i-0231cc03291ba78d601134e599d9241b-schwarzesloch-thumb-500x268.jpg
Ein schwarzes Loch krümmt den Raum besonders stark (Bild: AllenMC, CC-BY-SA 3.0)

Ein zweites Mißverständnis ist die Vorstellung des schwarzen Lochs als “Loch”. Im Prinzip ist sie nicht ganz falsch. Betrachtet man die klassischen populärwissenschaftlichen Vorstellungen der Raumzeit als zweidimensionale Fläche in der von Massen wie Sterne und Planeten Dellen erzeugt, dann kann man ein schwarzes Loch schon als eine Art “Loch” betrachten. Aber eigentlich ist es auch “nur” ein ehemaliger Stern der im Zuge einer Supernova-Explosion am Ende seines Lebens enorm stark komprimiert wurde. Dieses extrem dichte Objekt nennt man “Singularität” und es sitzt im Zentrum der Region die durch den Ereignishorizont definiert wird. Was aber passiert nun mit dem Material das den Ereignishorizont überschreitet? Fällt es sofort auf die Singularität?

Nicht unbedingt, meint Vyacheslav Dokuchaev von der russischen Akademie der Wissenschaften. Er hat eine (nicht peer-reviewte) Arbeit veröffentlicht die den spannenden Titel “Is there life inside black holes?” trägt. Ich bin kein Experte für die komplizierte Physik und Mathematik schwarzer Löcher und verstehe den Artikel daher nur teilweise – aber im Prinzip geht es um die Vorgänge in geladenen schwarzen Löchern. Denn neben seiner Masse ist die elektrische Ladung die zweite Eigenschaft die ein schwarzes Loch haben kann. Man wusste noch schon länger, dass hinter dem Ereignishorizont von rotierenden, geladenen schwarzen Löcher stabile periodische Bahnen um die Singularität möglich sind. Auf diesen Bahnen können dann beispielsweise Photonen um die Singularität kreisen anstatt auf sie zu stürzen.

Dokuchaev hat sich das nochmal genauer angesehen und probiert herauszufinden, welche Arten von Bahnen es dort nun wirklich gibt. Das ist wesentlich komplizierter als wenn man einfach nur nach stabilen Orbits um einen normalen Stern sucht. Beim schwarzen Loch muss man nicht nur die Gravitation berücksichtigen sondern auch die elektrische Ladung. Und man muss sich mit den seltsamen Dingen herumschlagen die das schwarze Loch mit den Dimensionen der Raumzeit anstellt. Da werden Raumdimensionen zu Zeitdimensionen (ein weiterer Grund, warum man nicht mehr aus dem Loch rauskommt wenn man mal drinnen ist: die entsprechende Richtung existiert einfach nicht mehr) und alles wird sehr verwirrend.

Ein schwarzes Loch hat aber neben dem Ereignishorizont noch einen weiteren, inneren Horizont. Der nennt sich “Cauchy-Horizont” und das “Lexikon der Astrophysik” (verfasst von Andreas Müller, Experte für schwarze Löcher) definiert ihn so:

“Eine Cauchy-Fläche ist eine Hyperfläche einer Raumzeit, die eine kausale Kurve exakt nur einmal schneiden kann.”

Tja… klingt simpel, nicht wahr? 😉 Worum es sich hier genau handelt ist mir selbst auch ein wenig unklar. Aber es handelt sich um eine Fläche, die ein wenig wie eine halbdurchlässige Membran funktioniert: man kann sie nur einmal durchqueren und dann bleibt man drinnen. Eine “kausale Kurve” ist eine “Weltlinie” als die Bahn, der ein Objekt durch die Raumzeit folgt. Innerhalb dieses Cauchy-Horizonts jedenfalls wird es wieder einfacher mit den stabilen Bahnen, meint Dokuchaev und die Verwirrung mit den vertauschten Dimensionen verschwindet. Dort konnte er nun einige stabile Bahnen um die Singularität finden auf der sich Planeten bewegen können – die aber natürlich völlig anders aussehen als die Planetenbahnen bei normalen Sternen. Mit den guten alten Keplerschen Gesetzen hat das hier nichts mehr zu tun:

i-908baf371b41b5a8c0422f0748676889-blackholeorbit-thumb-500x298.jpg

Das rote in der Mitte ist die Singularität, die blaue Kurve stellt die Bahn eines Planeten dar und die bunte Linie zeigt die Bahn eines Photons. Dokuchaev kommt in seiner Arbeit dann zu einer interessanten Schlussfolgerung:

“The advanced civilizations may live safely inside the supermassive BHs in the galactic nuclei without being visible from outside. The naked central singularity lluminates the orbiting planets and provide the energy supply needed for living. An dditional highlighting at night come from the eternally circulating photons.”

Eine fortgeschrittene Zivilisation könnte seiner Meinung nach also im supermassiven schwarzen Loch im Zentrum einer Galaxie leben. Die Singularität würde die Planeten dort beleuchten und ausreichend Energie liefern; zusätzlich gäbe es noch Licht von den Photonen, die die Singularität umkreisen.

Was ist davon zu halten? Nun ja… Dokuchaev nennt selbst einige mögliche Probleme. Man muss eventuell mit Kausalitätsproblemen klar kommen und es können durch die eingefangenen Photonen sehr hohe Energiedichten erzeugt werden. Und dann wäre da noch die enorm starken gravitativen Gezeitenkräfte. Ich persönlich frage mich ja, wie Planeten dort überhaupt hinkommen sollen. Wurden die komplett und unversehrt vom schwarzen Loch eingefangen? Haben sie sich dort aus diversen Bruchstücken neu gebildet? Hat die fortgeschrittene Zivilisation sie absichtlich dorthin transportiert? Und wie ist das mit den Effekten in der Nähe des Cauchy Horizonts? Andreas Müller schreibt im Lexikon der Astrophysik:

“Gelangt nun ein Beobachter auf einer Geodäte hinter den Cauchy-Horizont, so wird er Zeuge, wie die gesamte Geschichte der Außenwelt in Zeitraffer abläuft. Denn er erreicht eine Region unendlicher Blauverschiebung. Dummerweise wird er dann auch von einem energetisch betrachtet unendlichen Strahlungsblitz getroffen.”

Inwiefern Dokuchaevs Arbeit für ein besseres Verständnis der schwarzen Löcher interessant ist, kann ich nicht beurteilen, das sollen die Experten machen. Was die Frage nach Leben hinter dem Ereignishorizont angeht, halte ich den Artikel für zu spekulativ um hier irgendwelche Aussagen machen zu können. Die theoretische Möglichkeit stabiler Planetenbahnen um eine Singularität alleine erklärt noch nicht, wo solche Planeten herkommen sollen und wie Leben sich darauf entwickeln bzw. überhaupt leben kann. Das ist ein Thema, dass man vorerst wohl weiter den Science-Fiction-Autoren überlassen muss. In der hervorragenden “Gateway”-Serie von Frederick Pohl haben beispielsweise die “Heechee” aus Angst vor einem übermächtigen Feind all ihre Planeten in das innere des zentralen schwarzen Lochs unserer Milchstrasse verfrachtet um sich dort zu verstecken. Und Robert Forward erklärt in “Dragon’s Egg” (dt. “Das Drachenei”) und der Fortsetzung “Starquake” äußerst spannend und überzeugend, wie sich Leben auf der Oberfläche eines Neutronensterns (nach einem schwarzen Loch das zweitdichteste Objekt im Universum) entwickeln und abspielen könnte (Martin hat das Buch rezensiert). Vielleicht inspiriert ja die Arbeit von Dokuchaev ja den einen oder anderen Science-Fiction-Autor dazu, eine neue spannende Geschichte zu schreiben!


Flattr this

Kommentare (156)

  1. #1 Daniele
    18. April 2011

    Schön und gut, anschaulich erklärt und ich mag den Ansatz allgemeinen Irrglauben zu beseitigen. Aber dann hätte man der Anschaulichkeit zu Liebe auch sagen sollen, dass das gedachte schwarze Loch das die Sonne ersetzt und “nichts passiert” zwar gleich schwer sein kann, aber nicht gleich groß.
    Kurze Anmerkungen wie klein ein schwarzes Loch sein müsste um eben keinen veränderten (Gravitations)-Effekt auf die Erdbahn zu haben würden dem Leser sicher gefallen (also dem Leser populärwisschenschaftlicher Werke). Oder was würde passieren, wie weit würde sich ein Effekt einstellen, wenn das gedachte schwarze Loch die größe der Sonne hätte?

    Für promovierte Astronomen sind das Kleingkeiten, aber beim Leser bleibt es hängen wenn man z.B. sagt, dass dieses gedachte schwarze Loch nur die größe der Erde haben dürfe (z.B.) oder sowas…

    Ansonsten finde ich eh alle Artikel hier immer sehr spannend und gut aufgearbeitet.

  2. #2 Florian Freistetter
    18. April 2011

    @Daniele: “hätte man der Anschaulichkeit zu Liebe auch sagen sollen, dass das gedachte schwarze Loch das die Sonne ersetzt und “nichts passiert” zwar gleich schwer sein kann, aber nicht gleich groß”

    Hmm – versteh ich jetzt nicht.

    “Kurze Anmerkungen wie klein ein schwarzes Loch sein müsste um eben keinen veränderten (Gravitations)-Effekt auf die Erdbahn zu haben würden dem Leser sicher gefallen (also dem Leser populärwisschenschaftlicher Werke).”

    Naja – ein schwarzes Loch muss genau die Masse der Sonne haben, damit die Erdbahn sich nicht ändern. Ist die Masse größer oder kleiner, dann gibts Änderungen.

    “Oder was würde passieren, wie weit würde sich ein Effekt einstellen, wenn das gedachte schwarze Loch die größe der Sonne hätte?”

    Was meinst du genau? Ein schwarzes Loch hat per definitionem keine “Größe” sondern ist eine punktförmige Singularität. Die Ausdehnung des Ereignishorizonts hängt von der Masse ab? Meinst du das? Aber warum es jetzt interessant ist, ein SL zu betrachten dessen Ereignishorizont genau so ausgedehnt ist wie die Sonne, erschliesst sich mir nicht. Das, auf das es ankommt ist die Masse. Die bestimmt, wie die Erde sich bewegt.

  3. #3 Adrian
    18. April 2011

    “Gelangt nun ein Beobachter auf einer Geodäte hinter den Cauchy-Horizont, so wird er Zeuge, wie die gesamte Geschichte der Außenwelt in Zeitraffer abläuft. Denn er erreicht eine Region unendlicher Blauverschiebung. Dummerweise wird er dann auch von einem energetisch betrachtet unendlichen Strahlungsblitz getroffen.”

    Herr Serguei Krasnikov scheint dem zu widersprechen:
    “an observer falling into a Schwarzschild black hole will NOT see “the entire history of the universe””
    https://arxiv.org/abs/0804.3619v2 (freier Zugang)
    https://www.springerlink.com/content/g1172g282660vmr2/vv (peer-reviewed, kostenpflichtig)

  4. #4 Muriel
    18. April 2011

    Die Singularität könnte Planeten in einer Umlaufbahn beleuchten und mit Energie versorgen?
    Wäre das dann Restwärme vom Kollaps, oder würden da drin noch irgendwelche Reaktionen stattfinden? Sind darüber überhaupt einigermaßen belastbare Aussagen möglich, oder ist das wirklich reine Spekulation?

  5. #5 paule
    18. April 2011

    Der Ereignishorizontradius sollte 2Gm/c² betragen. Demnach wäre unser System bereits ein schwarzes Loch, wenn man m mit 1e53 kg ansieht. re wäre dann rund 15,7 mrd. LJ. Was merken wir davon? Außer grenzenloser Dummheit in diesem SL kann ich eigentlich nichts Besonderes feststellen.

  6. #6 frantischek
    18. April 2011

    “Ich persönlich frage mich ja, wie Planeten dort überhaupt hinkommen sollen.”

    Wenn ich mich recht erinnere hab ich einmal gelesen das auch sehr große Raumbereiche kollabieren könnten wenn in ihnen genug Masse angesammelt ist, nicht nur einzelne Sterne. Weiters stand da, das man wenn man sich innerhalb eines solchen Gebiets, z.b. auf einem Planeten befindet, erstmal vom Kollaps nicht viel merken würde.

    Kann sein das das in Greensteins “der gefrorene Stern” stand, weiß ich nicht mehr genau.

    P.S.: Jetzt ist es ja nicht mehr weit bis zu den alten Quasaren. Ich freu mich schon 😉

  7. #7 Bernd
    18. April 2011

    Das ist eine wirklich interessante Idee! In welcher Größenordnung würden diese Planetenbahnen auftreten?

  8. #8 KommentarAbo
    18. April 2011

  9. #9 Unwissend
    18. April 2011

    @KommentarAbo
    Du nimmst mit die Punkte aus den Gedanken

  10. #10 Name auf Verlangen entfernt
    18. April 2011

    Wir sollten doch dazu sagen, daß “Schwarze Löcher” eine wissenschaftliche Hypothese sind, die auf dem Standartmodell beruht, welches bekanntlich sicher nicht vollständig, höchstwahrscheinlich sogar schlicht falsch ist, bevor wir die Köpfe Deiner armen Leser weiter mit mythologisierenden Metaphern aus einer Märchenwelt verwirren. Die Frage ist also gar nicht, ob dort “Leben” existieren kann, sondern, ob “Schwarze Löcher” überhaupt existieren, mal abgesehen davon, daß die Naturwissenschaft z.Zt. genausoviel vom “Leben” weiß, die eine Nachteule von der Sonne.

  11. #11 nihil jie
    18. April 2011

    @frantischek

    ganze raumbereiche ? das erinnert mich an eine frage die ich hier mal gestellt habe. könnte ein raumbereich kollabieren in dem sich genügend “dunkle materie” angesammelt hat. ich meine nicht die MACHOs sondern DIE ominiöse dunkle materie. könnte sich in einer raumregion so viel davon ansammeln, dass sie den raum derart krümmt und dabei auch ein ereignishorizont ensteht ? sie wirkt doch hauptsächlich gravitativ. könnte es auch raumregionen geben die nicht durch das vorhanden sein normaler materie (sterne) kollabiert ist, sondern nur wegen einer enormen dichte der dunklen materie ?
    und noch eine frage… welche wechselwirkungen hat die dunkle materie auf sich selbst… also mit anderen dunkle materie partikeln… ausser der gravitation ?

    naja… fragen über fragen… aber eigentlich ist das thema hier nicht die dunkle materie sondern schwarze löcher und dem (un)möglichen leben darin 🙂

  12. #12 MartinS
    18. April 2011

    @FF
    Florian, es ist mir klar, dass das alles sehr hypothetisch ist, aber ein paar Fragen hätte ich doch.
    Planetenbahnen um ein BH:
    Die ‘Planetenbahn’, die die Zeichnung zeigt, scheint ja immer eine ‘Abweichung’ von der vorhergehenden zu haben. Damit könnte es doch überhaupt nicht zu einem ‘Freiräumen’ der Bahn kommen (wie bei ‘normalen’ Planeten) – also auch zu keiner Planetenentstehung!
    Könnte das fiktive Leben nicht eigentlich auch als ‘Bakterien’ (Miller’sche Ursuppe) auf den/der Planetenbahn entstehen? Ist für ‘Leben’ ein Planet denn zwingend notwendig? Könnte nicht dichte(re) Materie schon ausreichen?
    BH’s allgemein (nicht die der Damen!):
    Gibt es eine definierte Sternenmasse, bei der bei einem Kollaps des Sternes sicher ein BH entsteht?
    Vorausgesetzt meine Erinnerung ist korrekt, und alle BH’s ‘wachsen’ permanent (nehmen Materie auf), dann stellt sich mir die Frage, was geschehen würde, wenn keine frische Materie mehr zum BH käme. Würde sich ein BH selber auffressen? Gibt es ein (theoretisches) ‘Lebensende’ für so ein Ding?

  13. #13 rolak
    18. April 2011

    Lerne Physik, Markus. Aber vorher noch Syntax (Standard≠Standarte) und Semantik (Nachteule, ‘bis in’≠’nur in’)…

    Vielleicht für den Anfang bescheiden 40 Jahre wwwPause?

  14. #14 Daniele
    18. April 2011

    @Florian
    Wie gesagt, es hat viel damit zutun, an wen der Artikel grundätzlich gerichtet ist.
    Meine Fragen/Gedanken (und ich bin eben nicht vom Fach) waren, dass es ja schon verschieden “große/massereiche” SL gibt. Müsste in Deinem Bild im ersten Absatz das gedachte SL dann seinen Ereignishorizont im Bereich der Sonnengröße haben? Wäre damit die Gravitation genau so groß wie die der Sonne? Ich denke doch nicht, oder? Am EH überschreitet/unterschreitet (ja nachdem “von wo man kommt) die Fluchtgeschindigkeit c, dass heisst doch aber nicht dass man 1m vor dieser gedacht Grenze stehen kann.

  15. #15 BK
    18. April 2011

    Schwarze Löcher sind schon faszinierend…und ich gebe zu, ich liebe den alten Disney-Schinken mit Maximilian Schell 😉 (https://de.wikipedia.org/wiki/Das_Schwarze_Loch).

    Leider weiß ich wenig über Schwarze Löcher und habe daher einige vielleicht blödsinnige (“dumme” Fragen bitte ich zu entschuldigen^^) Fragen:
    Nur “aktive” Schwarze Löcher haben eine Akkretionsscheibe…ja/nein?
    Befindet sich der Ereignishorizont in der Akkretionsscheibe oder jenseits davon?
    Gibt es stabile Bahnen innerhalb einer Akkretionsscheibe oder fällt dort alles spiralförmig in Richtung Singularität?
    Ich habe mal vor Jahrzehnten (kann man mittlerweile sagen…bäh….ich werd alt) gelesen, dass es eine Theorie gebe, die besage, zu manchen Schwarzen Löchern gehöre ein “Weißes Loch”, aus dem alles wieder zutage trete, wass zuvor in die Singularität gezogen wurde….ist das irgendwie physikalisch möglich oder Humbug?

  16. #16 Name auf Verlangen entfernt
    18. April 2011

    @ rolak: besser nicht, dann würde ich ja an die “Singularität” glauben; da glaub ich lieber an die Jungfrau Maria.

  17. #17 Florian Freistetter
    18. April 2011

    @Adrian: “an observer falling into a Schwarzschild black hole will NOT see “the entire history of the universe””

    In dem Fall gehts aber um den Cauchy-Horizont; nicht den Schwarzschildradius. Was hat Krasnikov denn genau gemeint?

  18. #18 MartinS
    18. April 2011

    Ich begreife nicht, wieso M.T. behaupten kann, dass BH’s nur eine wissenschaftliche Hypothese sein?! Dass die Vorstellung davon, wie sie wirken, revidiert werden muss, kann ich hingegen akzeptieren.
    Immerhin ist er der lebende Beweis, dass man so ein SL im Kopf haben kann und dass es abstoßend wirkt!
    Wenn sich seine Gedanken im Kreis drehen – also um das besagte SL – dann ist es doch klar, dass durch Zeitinvarianz, Zeitumkehr und Ladungskonjugation der Quantenschaum ziemliche Blasen wirft. Und dank der Lorentz-Transformation wird ihm dann auch hinterher klar, warum ihm etwas vorher schon klar gewesen sein muss – oder so.
    @Daniele
    In dem von Florian verlinkten Artikel (“SLs sind keine Staubsauger”) erwähnt er, dass die Sonne auf einen Durchmesser von 3km schrumpfen würde. Keine Ahnung, wo der Ereignishorizont dann wäre.

  19. #19 Florian Freistetter
    18. April 2011

    @Markus: “Wir sollten doch dazu sagen, daß “Schwarze Löcher” eine wissenschaftliche Hypothese sind, die auf dem Standartmodell beruht, welches bekanntlich sicher nicht vollständig, höchstwahrscheinlich sogar schlicht falsch ist, bevor wir die Köpfe Deiner armen Leser weiter mit mythologisierenden Metaphern aus einer Märchenwelt verwirren”

    Ich weiß zwar, dass du dir gerne die Realität zurechtbiegst. Schwarze Löcher allerdings sind keine “Hypothese” sondern astronomische Objekte die tatsächlich beobachtet worden sind. Aber wahrscheinlich haben wir Astronomen nur wieder mal den Rest der Menschheit belogen, nicht wahr…

  20. #20 Florian Freistetter
    18. April 2011

    @MartinS: Also ein Stern muss etwa zehnmal so schwer wie die Sonne damit beim Kollaps am Ende des Lebens ein SL entsteht. Und wenn keine Materie mehr in der Nähe ist, dann macht das SL nix. Das sitzt dann einfach rum, gibt Hawking-Strahlung ab und in ein paar Trillionen Jahren oder so ist es verdampft 😉

  21. #21 MartinS
    18. April 2011

    @FF
    Danke!

  22. #22 Florian Freistetter
    18. April 2011

    @Daniele: Der Ereignishorizont bei einem schwarzen Loch mit der Masse der Sonne würde 2.9 Kilometer betragen.

    @BK: Die Akkretionsscheibe ist außerhalb des Ereignishorizonts und genau sie ist es, die ein SL aktiv macht. Aktiv heisst ja, dass Material drauf fällt. Was die weißen Löcher angeht, so sind die hauptsächlich hypothetisch: https://en.wikipedia.org/wiki/White_hole

  23. #23 Name auf Verlangen entfernt
    18. April 2011

    @ Florian: nein, nicht die Menschheit belügt ihr, sondern euch selbst: und der Wahheit halber solltest Du sagen, daß niemand jemals ein “Schwarzes Loch” beobachtet hat. Ihr habt eine Hypothese und sammelt eifrig “indirekte” Hinweise, deren Interpretation absolut offen ist. Würde man sonst hier mit Begriffen wie “Ereignishorizont” rumfuchteln? Derweil ist der Begriff “Schwarze Löcher” durch Realitäts-Verwischer bereits ein kollektives Mythologem, ebenso wie der Urknall und all eure anderen Hypothesen.

  24. #24 BK
    18. April 2011

    @ Florian
    Danke

  25. #25 rolak
    18. April 2011

    Ich befürchte, daß jmd den armen Markus erfolgreich belangt hat oder dergleichen – er tritt heute mit einer ausnehmend rattigen Laune und hochfrustaktiv auf. Siehe z.B. auch hier ff.

  26. #26 stl
    18. April 2011

    @all – (in Worten minus) MT
    Ist ein Schwarzes Loch jetzt eine Singularität oder nicht?
    Ein ideales SL wurde schließlich nicht beobachtet, oder?

  27. #27 Florian Freistetter
    18. April 2011

    @Markus: “Schwarzes Loch” ist die Bezeichung für ein Objekt bei dem eine bestimmte Masse inerhalb eines bestimmten Volumens konzentriert ist. Und genausowas beobachtet man. Z,B hier: https://www.scienceblogs.de/astrodicticum-simplex/2008/06/der-stern-zum-wochenende-s2.php

    Ansonsten werde ich aber nicht weiter mit dir über Astronomie diskutieren. Du diskutierst ja auch nicht mir mir über Astrologie, also passt das schon. Wenn du zu dumm bist, um zu verstehen, wie das in der Wissenschaft mit Theorie und Beobachtung, Hypothese und Bestätigung funktioniert, dann kann ich dir leider nicht weiterhelfen.

  28. #28 Florian Freistetter
    18. April 2011

    @alle: wer mehr über die Details zu schwarzen Löchern erfahren will dem empfehle ich dieses Buch hier: https://www.scienceblogs.de/astrodicticum-simplex/2010/09/wurmlocher-zeitmaschinen-und-ein-universum-dass-sich-selbst-erzeugt.php Da wird der Forschungsstand verständlich erklärt.

  29. #29 Mahalo
    18. April 2011

    Interessant, wieder was gelernt. Nett, mal dieses rumgetrolle hier. Seit wann ist Astrologie noch mal eine bewissene Wissenschaft? und erlaubt sich hier messbare Werte in Frage zu stellen. Hier werden gemessene Werte interpretiert. Was wird in der Astrologie doch gleich interpretiert? …

  30. #30 nihil jie
    18. April 2011

    @MT

    @ rolak: besser nicht, dann würde ich ja an die “Singularität” glauben; da glaub ich lieber an die Jungfrau Maria.

    also ohne glauben gehts bei dir wohl nicht… man kann sich auch mit dingen beschäftigen ohne, dass da ein glaube daran im spiel ist. nach deiner vorstellung müsste jeder physiker den physikalischen theorien und gesetzen mit glauben begegnen.

  31. #31 hallo
    18. April 2011

    Zitat: [Das ist ein Thema, dass man vorerst wohl weiter den Science-Fiction-Autoren überlassen muss.]

    Oh ja, ein guter zu diesem Thema ist: Sergej Snedow. https://www.amazon.de/Menschen-wie-G%C3%B6tter-Sergej-Snegow/dp/3360008383/ref=sr_1_5?ie=UTF8&qid=1303123357&sr=8-5

    Ein muss!

  32. #32 rolak
    18. April 2011

    Oh danke, nihil jie – den hatte ich zuerst verpaßt und war grad unterwegs, auf dieser thumben Projektion aus eigenen erbärmlichen Denkweisen herus ein wenig herumzuklopfen. Da kann ich meine Tippfinger schonen 🙂

  33. #33 nihil jie
    18. April 2011

    @MT

    ich lese dauernd von dir dass dies oder jenes, im weitestem sinne, blödsinn sein soll…
    dann lass bitte mal deine vorstellungen thesen oder gar theorien hören. erkläre uns mal bitte was an den theorien zum urknall und den schwarzen löchern usw… nicht stimmt… schildere uns mal deine lösungen.

  34. #34 nihil jie
    18. April 2011

    @rolak

    nix zu danken… versuche mich nur ein wenig sinnvoll zu beschäftigen. wir bekommen gerade eine lektion in Word *gähn* 😉

  35. #35 Mark
    18. April 2011

    Oben schreibst du, ein SL hat keine Größe, sondern ist eine punktförmige Singularität.
    Weiter unten schreibst du: “Schwarzes Loch” ist die
    Bezeichung für ein Objekt bei dem eine
    bestimmte Masse inerhalb eines
    bestimmten Volumens konzentriert ist.
    Ja was nun?

  36. #36 nihil jie
    18. April 2011

    @Mark

    bei einer singularität handle sich in unserem fall um eine zentralregion eines SL. das loch selbst… bis zum ereignishorizont, hat schon eine ausdehnung…

    @FF
    sorry, dass ich mich an die erklärung ran wage, obwohl ich nicht gefragt wurde 😉

  37. #37 Florian Freistetter
    18. April 2011

    @Mark: “Oben schreibst du, ein SL hat keine Größe, sondern ist eine punktförmige Singularität. Weiter unten schreibst du: “Schwarzes Loch” ist die Bezeichung für ein Objekt bei dem eine bestimmte Masse inerhalb eines bestimmten Volumens konzentriert ist. Ja was nun? “

    Naja, das ist erstmal kein Widerspruch. Auch in einer Singularität ist eine bestimmte Masse in einem bestimmten Volumen konzentriert. Aber ich weiß was du meinst, es geht um den Unterschied zwischen “schwarzes Loch” und “Singularität”. Ein SL nennt man oft alles, was hinter dem Ereignishorizont ist – wir können ja eh nicht dahinter schauen. Mit “Singularität” wird das eigentliche Ding bezeichnet, das hinter dem Horizont sitzt und das die eigentliche Masse des SL darstellt. Beobachten können wir die nicht, aber wenn wir – so wie beim oben verlinkten Stern S2 im Zentrum der Milchstrasse – beobachten können, dass sich innerhalb des Volumens X mindestens die Masse Y befinden muss, können wir eine Dichte berechnen und daraus schliessen, ob das massive Ding im Zentrum einen Ereignishorizont hat – also ein SL ist – oder nicht.

  38. #38 volki
    18. April 2011

    Mal eine vielleicht dumme Frage: Aber wieso ist es eigentlich klar, dass die Masse des SL “punktförmig” konzentriert ist und nicht z.B. innerhalb des Ereignishorizontes irgendwie “verschmiert” ist (zumindest lese ich das so im Beitrag und auch in den Kommentaren)?

  39. #39 Florian Freistetter
    18. April 2011

    @volki: “Aber wieso ist es eigentlich klar, dass die Masse des SL “punktförmig” konzentriert ist”

    Das ist nicht klar. Aber momentan ist keine Kraft bekannt, die den Kollaps bei einem schwarzen Loch aufhalten könnte. Das die Masse nicht wirklich auf einen Punkt kollabiert, ist wahrscheinlich; immerhin ist ein Punkt was abstraktes und kommt in der Realität normalerweise nicht vor. Vielleicht gibts noch eine unbekannte Kraft die den Kollaps irgendwann stoppt. Das wird sich aber erst klären lassen, wenn wir die Elementarteilchenphysik besser verstanden haben und z.B. herausgefunden haben ob die Stringtheorie stimmt oder nicht

  40. #40 knorke
    18. April 2011

    @nihil jie
    “ich lese dauernd von dir dass dies oder jenes, im weitestem sinne, blödsinn sein soll… dann lass bitte mal deine vorstellungen thesen oder gar theorien hören. erkläre uns mal bitte was an den theorien zum urknall und den schwarzen löchern usw… nicht stimmt… schildere uns mal deine lösungen.”

    Besser nicht. Wahrscheinlich kommt er nur zum Trollen vorbei um klicks auf seine Schwurbelseite zu generieren.

  41. #41 nihil jie
    18. April 2011

    @FF

    Das die Masse nicht wirklich auf einen Punkt kollabiert, ist wahrscheinlich; immerhin ist ein Punkt was abstraktes und kommt in der Realität normalerweise nicht vor.

    und was ist wenn die gesammte materie in der zentralregion in reine strahlung umgewandelt wurde ? bosonen dürfte es egal sein wie viel platz sie zur verfügung haben um sich zu konzentrieren ? oder ist in dem fall eine singuläre region zu viel des guten ?

  42. #42 Florian Freistetter
    18. April 2011

    @nihil: “und was ist wenn die gesammte materie in der zentralregion in reine strahlung umgewandelt wurde ?”

    Möglich ist viel 😉 Es gibt die Idee des Kugelblitz, ein SL, das nur aus Energie besteht anstatt aus Masse. Obs sowas wirklich gibt, ist wieder eine andere Frage.

  43. #43 nihil jie
    18. April 2011

    @FF

    die idee stammt sogar vom “meister” selbst… John Archibald Wheeler 😉 hat er auch nicht den namen “Schwarzes Loch” geprägt, nach dem er es satt hatte zum wiederholten male “ein in sich vollständig kollabiertes Objekt” zu sagen ?

  44. #44 volki
    18. April 2011

    @Florian: Mir ist klar, dass ein Punkt was abstraktes ist, darum habe ich auch punktförmig in ” ” gesetzt und mir einen sehr sehr kleinen Bereich vorgestellt z.B. eine Kugel mit Radius der Plancklänge oder so ähnlich. Aber auf jeden Fall Danke für die Antwort!

  45. #45 Robert
    18. April 2011

    Wenn ich mich auch noch einmischen darf: Da ist zunaechst noch das Video von S2, dem Stern, dem man beim Umrunden des Schwarzen Lochs im Zentrum der Milchstrasse zugesehen hat:

    https://www.eso.org/public/archives/videos/old_video/eso0226a.mpg
    hier die Erlaeuterung https://www.eso.org/public/videos/eso0226a/

    Und dann muss ich auch noch eine Runde klugscheissen, dass streng genommen die Singularitaet nicht im “Zentrum” Schwarzen Lochs ist, sondern in gewissem Sinn in seiner Zukunft: Was meine ich damit und woher kommt die Idee, sie sei im Zentrum?

    Um ein Objekt wie ein scharzen Loch zu beschrieben, bieten sich Kugelkoordinaten an: Dabei nimmt man nicht x,y und z, sondern zB r, was man sich als Radius, den “Abstand vom Zentrum” denken kann (aber was am Ende sich auch als falsch herausstellen wird), sowie zwei Winkel (zB Laenge und Breite). Und, ganz wichtig in diesem Geschaeft, auch noch t, die “am Ende leider auch nicht wirklich).

    Und in diesen Koordinaten sind beim schwarzen Loch (genau gesagt, beim Schwarzschild-Schwarzen Loch, dem kugelsymmetrischen, ungeladenen) der (Ereignis-)Horizont beim Radius R=2M (in Einheiten, wo ich die Newtonsche Gravitationskonstante G und die Lichtgeschwinidigkleit c zu eins gesetzt habe) und die Singularitaet bei r=0.

    Am Ereignishorizon passiert aber etwas seltsames: r und t tauschen ihre Rollen, innerhalb des Horizonts misst r die Zeit und t ist eine Raumrichtung! Der Zeitablauf in Richtung Zukunft ist nun das immer kleiner werden von r. Daher liegt r=0, die Singularitaet in der Zukunft, denn r=0 ist ein Zeitpunkt, kein Ort. Deswegen kann man im Schwarzschild-Schwarzen Loch auch der Singularitaet nicht entkommen. Man trifft sie genau so, wie man auch “heute Nachmittag 17h” nicht entkommen kann: Egal wohin ich gehe, irgendwann ist es 17h.

    Das klingt sehr seltsam, wie soll das gehen, Raum und Zeit vertauschen? Der Schluessel ist, sich zu ueberlegen, dass die Koordinaten nicht wirklich physikalisch sind, es sind bloss Namen oder Label, die wir an Punkte in der Raumzeit heften. Und wenn wir das mit bestimmten Vorurteilen tun (zB t ist wie folgt definiert: Die Raumzeit mit dem schwarzen Loch hat eine Symmetrie, weil es statisch ist, es aendert sich nicht. Diese Symmetrierichtung labeln wir mit t. Allerdings ist es eben nicht ueberall so, dass das eine zeitlich Symmetrie ist, innerhalb des Horizonts ist das eben eine raeumliche Symmetrie, so wie die Rotationssymmerie, die das schwarze Loch hat auch eine raeumliche Symmetrie ist). Letztendlich ist der Label “t” Menschengemacht. Man koennte auch andere Koordinaten verwenden, zB eine Koordinate T2, die immer in zeitlicher Richtung voranschreitet. Und dann waere eben die Singularitaet auch bei konstanter Zeit T2.

    Das erste Bild oben ist daher etwas irrefuehrend: Es ist an die Koordinate r angepasst (t ist gar nicht zu sehen). Eine Darstellung, die eher an die raum-zeitlichen Verhaeltnisse angepasst ist, ist das Carter-Penrose Diagramm:

    https://online.itp.ucsb.edu/online/colloq/hamilton1/oh/penrose_DM.html

    Da sieht man ein Dreick und ein auf der Spitze stehendes Quadrat. Dieses Bild ist in sofern ein besseres Abbild des schwarzen Lochs, als dass hier folgende Regeln gelten: Die Zeit laeuft ueberall von unten nach oben, links und rechts sind immer Raumrichtungen. Das gane ist etwas gestaucht, damit ein ganzes Universum auf ein Bild passt, aber die Winkel stimmen immer, insbesonder laufen Lichtstrahlen immer auf schraegen Linien im 45 Grad Winkel.

    Der Bereich ausserhalb des Horizonts ist das innere des Quadrats, der Bereich innerhalb des Horizonts das Dreieck. Ausserdem sind da noch duennere, gekruemmte Linien gemalt, im Quadrat von unten nach oben laufend, im Dreieck eher horizontal. Das sind Linien konstanten r’s. Man kann deutlich sehen, dass im Dreieck, wenn man mit der Zeit nach oben laeuft, man eine r-Linie nach der anderen ueberschreitet und schliesslich bei der Singularitaet r=0 ankommt, die die Zickzackline oben am Bildrand ist. (das ist nix zickzack, in Wirklichkeit ist das eine horizontale Linie, das zickzack deutet nur an, dass hier die Kruemmung unendlich gross wird). Man sieht aber insbesondere auch, dass die Singularitaet nicht nur ein Punkt, sondern eine ganze Linie ist. In der Tat hat die Singularitaet eine raumliche Ausdehnung! Dies ist anders als bei der Erde, bei der der Mittelpunk (R_erde=0) nur ein Punkt ist.

    Man kann natuerlich auch ausserhalb des Horizonts bleiben und beliebig lang warten. Dann landet man auch in der Zunkunft und zwar in der obersten Ecke des Quadrats.

    Fuer rotierende oder geladene Schwarze Loecher ist das ganze etwas komplizierter, hier konsultiere man die Wikipedia https://en.wikipedia.org/wiki/Penrose_diagram

    Darauf bezieht sich auch der oben erwaehnte Artikel: Da kann man in der Tat die Singularitaet vermeiden, da sie da nicht immer in der Zukunft liegt.

  46. #46 stl
    18. April 2011

    @nihil jie und volki
    Vielen Dank für die Fragen (Florian selbstverstänndlich für die Antworten), denn jetzt ist meine von irgendwo oben beantwortet. Ich lerne das noch, mich korrekt auszudrücken 🙂

  47. #47 Martin
    18. April 2011

    Irgendwie habe ich das Gefühl, dass die Frage “Gibt es Leben in schwarzen Löchern?” genau so sinnvoll zu stellen ist, wie die Frage, ob es Leben im Erdkern gibt. Und die Antwort ist auch ähnlich:
    “Eigentlich nein, aber wenn die physikalischen Bedingungen stimmen würden, ja.”

  48. #48 Martin
    18. April 2011

    Irgendwie habe ich das Gefühl, dass die Frage “Gibt es Leben in schwarzen Löchern?” genau so sinnvoll zu stellen ist, wie die Frage, ob es Leben im Erdkern gibt. Und die Antwort ist auch ähnlich:
    “Eigentlich nein, aber wenn die physikalischen Bedingungen stimmen würden, ja.”

  49. #49 Martin
    18. April 2011

    Irgendwie habe ich das Gefühl, dass die Frage “Gibt es Leben in schwarzen Löchern?” genau so sinnvoll zu stellen ist, wie die Frage, ob es Leben im Erdkern gibt. Und die Antwort ist auch ähnlich:
    “Eigentlich nein, aber wenn die physikalischen Bedingungen stimmen würden, ja.”

  50. #50 Andreas
    18. April 2011

    Ich dachte immer, die Masse in einem SL nicht punktförmig sein kann, weil dann die Rotation des SL unendlich groß werden, die Gravitation aufheben und die Singularität damit “nackt”, also ohne Ereignishorizont sein würde… hab ich da was falsch verstanden?

  51. #51 Ex-Esoteriker
    18. April 2011

    Sehr schöner Artikel, aber da ist wirklich noch soo viel Forschungsbedarf, wenn jetzt sogar die Frage nach Leben in einem SL gefragt wird.

  52. #52 Name auf Verlangen entfernt
    18. April 2011

    @ Florian: Nur mal so kurz Dein eigener O-Ton, wie sich da der Ereignishorizont schleichend verschiebt:

    Heute: ” … Schwarze Löcher allerdings sind keine “Hypothese” sondern astronomische Objekte die tatsächlich beobachtet worden sind. “

    Vor´n paar Tagen: “Direkt beobachtet werden konnte so ein schwarzes Loch aber natürlich noch nicht. Dort, wo sich das Zentrum unserer Milchstrasse befindet konnte allerdings eine starke Radioquelle ausgemacht werden. Dieses Objekt – Sagittarius A* (SgrA*) – ist vermutlich das schwarze Loch.”

  53. #53 Florian Freistetter
    18. April 2011

    @Markus: “Nur mal so kurz Dein eigener O-Ton, wie sich da der Ereignishorizont schleichend verschiebt”

    Tja, sinnerfassendes Lesen ist halt nicht jedermanns Sache. Aber wenn du glücklicher damit bist, dann ändere “beobachtet” in “nachgewiesen”. Aber du wirst ja sowieso weiterhin die Realität nach deinen Auffassungen zurechtbiegen, also ists auch wieder egal.

  54. #54 Fission Chips
    18. April 2011

    es ist durchaus möglich lebendig den schwarzschildradius zu passieren. bei sehr grossen schwarzen löchern sind die gravitationsunterschiede schwach genug um nicht direkt zerrissen zu werden…

    die blauverschobenen photonen sind natürlich ein anderes problem

  55. #55 Bullet
    18. April 2011

    LOL … MT kennt den Unterschied zwischen stellaren BH und SMBH nicht.
    Weiaweiaweia.

  56. #56 Niels
    18. April 2011

    Es funktioniert ausdrücklich nur bei rotierenden, geladenen schwarzen Löchern, richtig?

    Rotierende Schwarze Löcher haben gar keine Punkt-Singularität, sondern eine sogenannte Ringsingularität.
    Das ist ne ziemlich komplizierte Sache, die ich auch nicht richtig verstehe. 😉
    https://www.wissenschaft-online.de/astrowissen/lexdt_r05.html#ring

    Bei der Interpretation des Ringradius muss man aufpassen: bei einer Visualisierung der Krümmungsinvarianten wird klar, dass der Ring keine Ausdehnung hat. Die Ringsingularität befindet sich immer innerhalb des inneren Horizonts, des so genannten Cauchy-Horizonts. Die Ringsingularität sitzt wie bei Schwarzschild bei r = 0, hat aber dennoch einen anderen Charakter. Diese Eigenschaft ist am schwierigsten zu verstehen und erfordert eine genaue Analyse der Singularitätenstrukturen in den richtigen Koordinaten[…]
    Offensichtlich lässt sich nur erahnen, dass die intrinsische Kerr-Singularität vollkommen wesensverschieden von der intrinsischen Schwarzschild-Singularität ist.

    Das größte Problem liegt aber doch bei der Ladung. Wie will das (intelligente Leben) im Loch verhindern, dass geladene Teilchen ins Loch fallen?
    Dazu müsste man das Loch völlig isolieren.
    Sobald nämlich Teilchen ins Loch fallen, verändert sich die Ladung des Loches wodurch sich auch die stabilen Bahnen verändern.
    Im schlimmsten Fall kommt es sogar zu einem Ladungsausgleich und das schwarze Loch ist wird ungeladen, dann gibt es anscheinend gar keine stabilen Bahnen mehr.
    Soweit ich weiß, gehen Astronomen davon aus, dass es keine geladenen schwarzen Löcher gibt. Durch einfallende einfallende geladene Materie sollte sich die Ladung nämlich in kurzer Zeit ausgleichen.

    Schon irgendwie blöd, wenn man in so einem Fall dann nicht umziehen kann.

  57. #57 nihil jie
    18. April 2011

    meine frage wäre auch, ob es denn überhaupt NICHT-rotierende SL gibt ?

  58. #58 Andreas P.
    18. April 2011

    @MartinS

    Planetenbahnen um ein BH:

    Schöner freudscher Verleser .. im ersten Anlauf hab ich Planetenbahnen um einen BH gelesen, mit dem passenden Gedanken “Planetenbahnen?! Mensch, das müssen aber gewaltige …” 🙂

    @FF
    Eigentlich wollte ich ja nachfragen wie man sich das praktisch vorstellen muss mit solchen stationären Planetenbahnen “innerhalb” des SL .. auch hinter dem Ereignishorizont sollte doch die Gravitation genau so wirken wie ausserhalb, und jeder Körper auf einer stationären Bahn müsste dem doch durch entsprechende Ratation entgegenwirken, oder? Mit anderen Worten, das geht doch nur bei entsprechend großen SL, wo der Planet entsprechend weit weg von der Singularität seine Bahn haben kann? Und besonders gross darf der Planet auch nicht sein, ansonsten müsste er doch durch die unterschiedliche Gravitation an zu- und abgewandter Seite zerissen werden, oder verschätze ich mich da?

    Aber dann las ich Robert und seine vertauschten Raum-Zeit-Koordinaten, und jetz bin ich wirr im Kopf, das muss ich erst mal sortieren 🙂

  59. #59 nihil jie
    18. April 2011

    @Andreas P.

    Aber dann las ich Robert und seine vertauschten Raum-Zeit-Koordinaten, und jetz bin ich wirr im Kopf, das muss ich erst mal sortieren 🙂

    ich denke, dass Du nicht der einzige bist der gerade am sortieren ist 😉

  60. #60 Christian Berger
    18. April 2011

    Hmm, Moment mal, wenn ich jetzt einen Staubsauger in den interplanetaren Raum “stelle” und ihn einschalte, und irgendwie das Überhitzungsproblem löse, so macht er doch genau das selbe wie schwarze Löcher, er zieht mit seiner Gravitation Materie an. Saugen kann er ja im Vacuum nicht.

    Wenn sich also jetzt ein schwarzes Loch in einen Staubsauger gleicher Masse verwandeln würde, so würde sich höchstens in der direkten Umgebung etwas ändern.

  61. #61 Florian Freistetter
    18. April 2011

    @Christian: “Wenn sich also jetzt ein schwarzes Loch in einen Staubsauger gleicher Masse verwandeln würde, so würde sich höchstens in der direkten Umgebung etwas ändern. “

    Ja, aber die Idee des “saugens” impliziert ja immer eine stärker werdende Kraft. Ein schwarzes Loch übt – so wie alles andere auch – Gravitationskraft aus. Die Objekte in seiner Umgebung sind entweder weit weg/schnell genug um nicht drauf zu fallen oder sie fallen drauf. Aber die Gravitiationskraft ändert sich (erstmal) nicht. Damit die Erde von einem SL das die Sonne ersetzt “angesaugt” wird, müsste sich die Masse dieses SL stetig erhöhen damit auch die Anziehungskraft immer größer wird. Ansonsten würde die Erde einfach weiter ihre Runden drehen.

  62. #62 noch'n Flo
    18. April 2011

    @ Andreas P.:

    “Planetenbahnen?! Mensch, das müssen aber gewaltige …”

    Naja, bei so mancher Frau möchte man (nach diversen Operationen) schon glauben, dass die Dinger den Raum krümmen…

  63. #63 nihil jie
    18. April 2011

    @noch’n Flo

    lästere nicht über cyborgs *gg

  64. #64 Nele
    18. April 2011

    Pöh. Ihr könnt alle schreiben, was ihr wollt – seit meinen Kindheitstagen weiß ich, dass Dr. Reinhardt mit einem großen, roten Schweberoboter verschmolzen in einem strudelförmigen schwarzen Loch lebt.

    Jawoll.

  65. #65 Alexandra
    18. April 2011

    Guten Abend,
    mich beschäftigt seit längerem eine Verständnisfrage zu Schwarzen Löchern:
    Es ist doch so, dass die Raumzeit, je näher man einem Schwarzen Loch kommt, immer stärker gekrümmt ist, richtig? Bedeutet das nicht, dass es unmöglich ist, jemals ein Schwarzes Loch zu erreichen, da ab einer gewissen Annäherung die Zeit für den Beobachter praktisch stehenbleibt (oder bis ins Unendliche gedehnt wird)? Das würde doch dann bedeuten, dass es nicht nur unmöglich ist, dem Schwarzen Loch jemals zu entkommen, sondern auch, von außen zu dem Schwarzen Loch zu gelangen. Oder verstehe ich das falsch? Kann das schwarze Loch dann überhaupt Materie von außen aufnehmen?
    Ich hoffe, die Frage ist nicht zu laienhaft…
    Viele Grüße
    Alexandra

  66. #66 Lola
    18. April 2011

    ***Gibt es Leben in schwarzen Löchern?****

    Maulwurf.

  67. #67 Schlotti
    18. April 2011

    @Alexandra:

    Es ist doch so, dass die Raumzeit, je näher man einem Schwarzen Loch kommt, immer stärker gekrümmt ist, richtig? Bedeutet das nicht, dass es unmöglich ist, jemals ein Schwarzes Loch zu erreichen, da ab einer gewissen Annäherung die Zeit für den Beobachter praktisch stehenbleibt (oder bis ins Unendliche gedehnt wird)? Das würde doch dann bedeuten, dass es nicht nur unmöglich ist, dem Schwarzen Loch jemals zu entkommen, sondern auch, von außen zu dem Schwarzen Loch zu gelangen.

    Wenn ich die Theorie der schwarzen Löcher richtig verstanden habe, bleibt die Zeit für einen externen Beobachter durchaus nicht stehen. Aus Sicht eines externen Beobachters fällt ein Objekt, welches sich dem Ereignishorizont zu sehr nähert, in das schwarze Loch hinein. Dies ist, wenn der Ereignishoritont passiert wird, unumkehrbar.

    Für denjenigen, der das ausprobiert, mag tatsächlich die Zeit “stehenbleiben”. Aber nicht für einen ausserhalb befindlichen Beobachter.

    Ich persönlich kann (mangels Fachwissens) nicht einschätzen, was mit einem Objekt passiert, welches von einem schwarzen Loch “absorbiert” wird. Aber für einen Beobachter ausserhalb des Ereignishorizontes stürzt ein solches Objekt schlicht in das schwarze Loch hinein und ist weg.

    Für einen ausserhalb des Ereignishorizontes befindlichen Beobachter stürzt ein Objekt in das schwarze Loch und ist damit keiner weiteren Beobachtung zugänglich.

    Wie die “Bewohner” eines Objektes, welches in ein schwarzes Loch stürzt, dies wahrnehmen, zumal am Ereignishorizont die Zeit, so wie wir sie wahrnehmen, aufhört zu existieren, entzieht sich meinem Verständnis.

    Immerhin, werte Alexandra, dürfen wir beide zuversichtlich sein, hier von besser informierten Leuten, als wir selber sind, gute Antworten zu erhalten.

    Fragen werden hier nämlich (das ist ein Erfahrungswert) immer gerne beantwortet.

  68. #68 schlappohr
    18. April 2011

    @Alexandra

    “Es ist doch so, dass die Raumzeit, je näher man einem Schwarzen Loch kommt, immer stärker gekrümmt ist, richtig?”

    So ist es laut aktuellem Stand der Physik.

    “Bedeutet das nicht, dass es unmöglich ist, jemals ein Schwarzes Loch zu erreichen, […]”

    Als ein Beobachter, der in ein SL fällt, ändert sich an Deinem subjektiven Zeitgefühl nichts. Du näherst Dich dem Ereignishorizont und überquerst ihn. (Was danach passiert, weiß noch niemand so genau.) Wenn Du währenddessen jedoch zurückschaust, siehst Du, wie die Zeit in dem Universum, dass Du gerade verlässt, zu rasen beginnt. Du siehst, wie sich Galaxien drehen und Sterne in Sekunden entstehen und verglühen. Du siehst, wie Dinosaurier entstehen und eine Sekunde später aussterben.
    Ein anderer Beobachter, der Deinen Fall aus sicherer Entfernung verfolgt, sieht, wie alles was Du tust, endlos lange dauert. Dein Winken mit dem Taschentuch zum Abschied dauert Jahrmillionen.
    Das ist es, was die ART sagt: Es gibt eine absolute Zeit. Jeder kann den Lauf der Zeit nur aus seiner subjektiven Sicht beurteilen, und wenn zwei Beobachter zu unterschiedlichen Ergebnissen kommen, so haben beide recht.
    George Greenstein hat das in seinem Buch “Der gefrorene Stern” sehr eindrucksvoll beschrieben: Ein SL beginnt erst dann zu existieren, wenn Du hineinfällst. Von außen betrachtet, bleibt es das SL in seiner Entstehung irgendwo stehen. Aus der Nähe betrachtet bildet es eine Singularität.

    So hab ich es zumindest verstanden… ich bin auch nur ein Laie (was mich ehrlich gesagt manchmal gewaltig nervt). Also wenn ich hier Unsinn erzähle, korrigiert mich bitte (der Aufruf geht nur an Nicht-Astrologen).

  69. #69 Schlotti
    18. April 2011

    Nachtrag:

    Ich schrieb:Fragen werden hier nämlich (das ist ein Erfahrungswert) immer gerne beantwortet. Ich möchte ergänzen:

    Insofern es sich nämlich um sinnvolle, höflich formulierte und nachvollziehbare Fragen handelt.

    (Du scheinst hier neu zu sein, Alexandra, deshalb der Hinweis darauf, dass hier manchen Idioten gegenüber ein recht heftiger Wind weht. Deine Frage ist durchaus nicht laienhaft, sondern nachvollziehbar und berechtigt. Manche Leute, die hier posten, haben allerdings bestenfalls einen IQ=Außentemperatur und spielen sich hier trotzdem als allwissend auf. Lies mal die diversen Posts irgendwelcher Astrologen, und du weisst, was ich meine…))

    Wie schon gesagt, ich bin zuversichtlich, dass wir beide – bezogen auf schwarze Löcher – demnächst schlauer sein werden, als wir es jetzt sind.

    Warten wir es ab…

    MfG,

    Schlotti

  70. #70 Odysseus
    19. April 2011

    @Alexandra und Schlotti: Wenn man vom “Reinfallen” ins Schwarze Loch redet, meint man meistens das Passieren des Ereignishorizontes, also der Grenze, die man von der Innenseite nicht mehr überwinden kann. Nun ist es aber so, dass dieser Punkt in der Raumzeit lokal nicht besonders ausgezeichnet ist: Genausowenig wie die Datumsgrenze oder der Äquator auf der Erde mit einer roten Linie markiert sind, bemerkt man als Astronaut, dass man gerade den Horizont überquert, also “in” das SL fällt.
    Die Raumzeitkrümmung ist an diesem Punkt — der je nach Masse einige Kilometer oder auch viel weiter von der Singularität entfernt ist — grundsätzlich endlich. Die Gezeitenkräfte zerreißen zwar jede Materie, reinfallen können deine Überreste aber trotzdem problemlos. Erst im Mittelpunkt des SL wird die Krümmung (wie die Massendichte) unendlich, oder vornehm: singulär.

  71. #71 JD
    19. April 2011

    Ich gestehe, dass ich jetzt noch nicht alle Kommentare gelesen habe. Aber ergänzend zum ersten Drittel will ich noch auf ein weiteres Buch hinweisen in dem Leben in / auf schwarzen Löchern durchgespielt wird: Steven Baxter: Zeit

    mfg

  72. #72 rolak
    19. April 2011

    Ha, er ist geständig 🙂
    Zum Horizonteffekt bzgl Zeitempfinden würde die gute alte ‘Kyrie’ von Poul Anderson passen (erschienen in Ullstein SF-Stories 51, 1975)

  73. #73 Moss
    19. April 2011

    @Odysseus:

    Die Gezeitenkräfte zerreißen zwar jede Materie

    Sicher immer? Gezeitenkräfte werden ja dadurch erzeugt, dass die lokale Raumzeit um Gravitationsquellen wie Schwarzen Löchern (SL) nicht flach ist, so dass zwischen SL-zu- und -abgewandten Enden von ausgedehnten (nicht punktförmigen) Objekten unterschiedliche Gravitationsbeschleunigungen auftreten. Bei steigender Masse des SL steigt auch der Durchmesser des Ereignishorizonts, damit sollte das Krümmungsdifferential dort sinken, also sollten bei genügend großer Masse im SL genügend kleine Objekte zumindest den Ereignishorizont unbeschadet durchqueren können – oder irre ich da?

    Gut, den Astronauten zerrupft’s dann halt weiter drin im Gravitationsschacht … 😉

  74. #74 Alexandra
    19. April 2011

    Vielen Dank an Schlappohr, Schlotti und Odysseus!
    Dann ist es also so, dass die Problematik mit der Zeitdilatation erst dann auftritt, wenn das einfallende Objekt den Ereignishorizont bereits überschritten hat und für einen Beobachter von außen gar nicht mehr sichtbar ist. Gut! Sonst hätte sich ja auch die Frage gestellt, ob das Schwarze Loch nicht ‘verdampft’ (siehe Hawkins-Srahlung oben), bevor es sich überhaupt neue Materie von außen zuführen kann…
    Nichts desto trotz kann ich nicht umhin, mir das Leben im Schwarzen Loch (bezw. innerhalb des Ereignishorizonts des Schwarzen Loches) ziemlich ungemütlich vorzustellen: irgendwo muss ja die ganze Strahlung/Wärme/Licht hin, die nicht entkommen kann. Andererseits kann aus unserem Universum ja auch keine Strahlung/Wärme/Licht entkommen und uns geht es hier ja trotzdem ziemlich gut.
    Zu schade, dass die meisten dieser Fragen zu unseren Lebzeiten wohl nicht mehr geklärt werden können, das ist so spannend!!

  75. #75 MartinS
    19. April 2011

    Nachdem ich schon am Text von @Robert gescheitert bin (was nicht an Robert liegt!) und mir die Zeitdilatation ebenfalls schwer im Magen liegt:
    Ein Partikel, das irgendwie auf eine Kreisbahn um das BH gelangt ist (hinter dem Ereignishorizont) müsste doch eigentlich mit ‘unglaublicher’ Geschwindigkeit um das BH kreisen, damit seine Bahn erhalten bleibt. Da aber doch die Zeit hinter dem Ereignishorizont eine andere ‘Geschwindigkeit’ hat, als bei uns, würde mich interessieren, ob sich die Orbitalgeschwindigkeit dann wieder auf ‘Unterlicht’ reduzieren würde?
    Oder anders gefragt:
    Wenn die Sonne ein BH wäre – mit dem Ausmaßen unserer heutigen Sonne – wie schnell müsste die Erde sein, um ihre Kreisbahn halten zu können?
    Hat irgend jemand verstanden, was ich wissen möchte?
    Gibt die Mathematik das überhaupt her (die Physik wohl eher nicht!)

  76. #76 MartinB
    19. April 2011

    @MartinS
    Wenn die Sonne durch ein BH mit Sonnemasse ersetzt würde, dann würde sich hier gravitativ nichts ändern.

    Wenn die Sonne durch ein supermassives BH ersetzt würde, dann müsste man entsprechend schneller kreisen – dicht oberhalb des Ereignishorizonts sind soweit ich weiß keine stabilen Bahnen mehr möglich.

    Das mit dem “hinter dem Ereignishorizont” scheint mir ein Missverständnis zu sein – ein Planet auf einer Kreisbahn ist ja immer außerhalb des (kugelförmigen) Ereignishorizonts. Wenn du von “oben” auf die Bahn draufguckst, siehtst du, dass sie ganz symmetrisch ist

  77. #77 Niels
    19. April 2011

    @Moss
    Richtig, Astronauten können das Überschreiten des Ereignishorizontes bei schweren schwarzen Löchern problemlos überleben.
    Ich denke, so hat Odysseus das aber auch gemeint, sonst hätte er nicht geschrieben, dass man das Überschreiten des Horizontes gar nicht bemerkt, oder?

    @MartinS
    Warum müsste sich das Teilchen denn mit ‘unglaublicher’ Geschwindigkeit bewegen?
    Teilchen im Raum können sich nie schneller als c bewegen.
    Die Raumzeit um ein rotierendes Loch rotiert allerdings mit.

    Wenn die Sonne ein BH wäre – mit dem Ausmaßen unserer heutigen Sonne – wie schnell müsste die Erde sein, um ihre Kreisbahn halten zu können?

    Soll das BH die Masse der Sonne haben?
    Denn bleibt die Erde einfach auf ihrer Kreisbahn.

    Oder soll der Ereignishorizont den Radius der Sinne haben?
    Dann wäre die Masse viel größer als die Masse der Sonne, die Erde müsste sich schneller um die Sonne drehen.

  78. #78 Niels
    19. April 2011

    Nachtrag:
    @MartinS
    Das, was ich oben behauptet habe, stimmt so allgemein nur für ungeladene, nichtrotierende schwarze Löcher (schwarzschild black holes). Im Artikel geht es allerdings um geladene, rotierende BH.
    Allgemein wird das dann je nach Ladung und Rotation ziemlich kompliziert.

    @MartinB
    Bei rotierenden und/oder geladenen schwarzen Löchern ist der Ereignishorizont nicht mehr kugelförmig.

  79. #79 MartinS
    19. April 2011

    Danke für eure Antworten, aber meine Befürchtungen haben sich bestätigt! Ich habe mich zu schwurbelig ausgedrückt.
    Grundsätzlich bezog ich mich auf Florian’s Text.

    Man wusste noch schon länger, dass hinter dem Ereignishorizont von rotierenden, geladenen schwarzen Löcher stabile periodische Bahnen um die Singularität möglich sind.

    @MartinB
    Basierend auf dem angeführten Zitat könnten doch stabile Bahnen möglich sein?! Ich sprach ja auch absichtlich nur von Partikeln (und nicht Planeten) auf so einer hypothetischen Bahn.

    @Niels
    Hattes Du nicht selbst – ich glaube bei Ludmila (und bei MartinB)– geschrieben, dass das Licht hinter dem Ereignishorizont des Universums >C wäre? Bzw, dass sich die Galaxien/Sterne irgendwann der Beobachtbarkeit entziehen würden?
    Daher zumindest stammt meine Annahme, dass es bei SLs vergleichbar wäre. Aus Deiner Bemerkung, dass die Raumzeit mit rotiert, schließe ich, dass ‚im Einflussbereich’ / hinter dem Ereignishorizont wieder ‚unsere’ Naturgesetze gelten (also nichts >C)?
    Basierend auf Florian’s Aussage, dass die Sonne im Falle eines Kollabierens auf ca 3km zusammenfallen würde (der Ereignishorizont) war meine Frage, wie schnell die Kreisbahngeschwindigkeit der Erde wäre, wenn der Ereignishorizont die Ausmaße unserer heutigen Sonne hätte (größere Masse ebenfalls). Nur als Gedankenspielerei! Wenn es eine Formel für Dummies gibt, dann rechne ich auch selbst.
    Mit „v=2pir/T komme ich nicht weiter. Da fehlt doch die Masse?!Da fehlt eigentlich ALLES! 🙁

  80. #80 Niels
    19. April 2011

    @MartinS

    Hattes Du nicht selbst – ich glaube bei Ludmila (und bei MartinB)– geschrieben, dass das Licht hinter dem Ereignishorizont des Universums >C wäre? Bzw, dass sich die Galaxien/Sterne irgendwann der Beobachtbarkeit entziehen würden?

    Objekte können sich nicht schneller als mit c im Raum bewegen.
    Objekte können sich aber mit Geschwindigkeiten höher als c entfernen.
    Im Fall der Expansion des Universums kommt die Fluchtgeschwindigkeit der Galaxien nicht durch eine Bewegung im Raum, sondern durch die Expansion des Raumes selbst zu Stande.
    (Argh, nicht die beste Formulierung.)

    Daher zumindest stammt meine Annahme, dass es bei SLs vergleichbar wäre.

    Öh, ein Objekt innerhalb des Ereignishorizontes ist für einen Beobachter außerhalb des Ereignishorizontes gar nicht sichtbar. Es entzieht sich schon beim Fallen durch den Horizont der Beobachtbarkeit.
    Lokal bewegt sich das Objekt für einen Beobachter auf diesem Objekt immer mit einer Geschwindigkeit kleiner als c.

    Aus Deiner Bemerkung, dass die Raumzeit mit rotiert, schließe ich, dass ‚im Einflussbereich’ / hinter dem Ereignishorizont wieder ‚unsere’ Naturgesetze gelten (also nichts >C)?

    Na ja, wenn man solche Rechnungen macht, geht man schon davon aus, dass auch in einem schwarzen Loch noch die ART gilt. Sonst könnte man schließlich gar nichts rechnen.
    Die Raumzeit rotiert immer um einen rotierenden Körper mit. Die Raumzeit rotiert also sogar mit der Sonne bzw. mit der Erde, da beide eine Eigenrotation besitzen.
    Im Fall der Erde ist das ein gerade noch messbarer Effekt.
    https://de.wikipedia.org/wiki/Gravity_Probe

    war meine Frage, wie schnell die Kreisbahngeschwindigkeit der Erde wäre, wenn der Ereignishorizont die Ausmaße unserer heutigen Sonne hätte (größere Masse ebenfalls). Nur als Gedankenspielerei! Wenn es eine Formel für Dummies gibt, dann rechne ich auch selbst.

    Na ja, zuerst musst du den Radius der Sonne kennen.
    Dann berechnest du über den Schwartzschild-Radius die Masse M des schwarzen Loches.
    https://upload.wikimedia.org/math/1/b/0/1b00088b473e3e870dadf982ce663ebb.png
    (Für den Schwartzschild-Radius den Sonnenradius einsetzen.)
    Wenn du es dir dann einfach machen willst, nimmst du die Erbahn als Kreisbahn an und setzt Gravitionskraft gleich Zentrifugalkraft und löst nach der Geschwindigkeit auf.

    Allerdings hat das mit dem Thema des Artikels überhaupt nix zu tun, da geht es ja um Bahnen innerhalb des Ereignishorizontes.
    Da muss man dann dringend die ART berücksichtigen, also mit der passenden Metrik rechnen usw. Das ist dann verdammt kompliziert.

  81. #81 MartinB
    19. April 2011

    @MArtinS
    Entschuldigung, ich hatte dich völlig missverstanden.
    Niels hat wie üblich alles sauber erklärt, soweit ich sehen kann.

  82. #82 MartinS
    19. April 2011

    @Niels & MartinB
    Danke für eure Antworten (besonders Niels!). Das kommt eben dabei heraus, wenn sich ein Laie für derartig abgefahrene Sachen interessiert 🙂
    Aber so schlimm wie MT bin ich noch nicht, oder?!
    Danke euch 🙂 !

  83. #83 T'Pau
    20. April 2011

    Um ein wenig zur “Überlichtgeschwindigkeit” beizutragen.

    Das folgende stammt aus dem Buch “Kleines 1×1 der Relativitätstheorie” – eigentlich als Geschenk für meine kleine Cousine, die jetzt schon in der Unterstufe weiss, dass sie Physik studieren will (mal schauen was draus wird ;-)) und das Buch versteht trotz Oberstufenmathematik (ja, da kann man Stolz heraus hören :-)), erstaunt mich das Buch selbst, weil es durchaus komplexe Sachverhalte erstaunlich anschaulich darstellen kann.

    Betreffs “Überlichtgeschwindigkeit” in Schwarzen Löchern: Gegeben sind rein statistische Punkte, also hypothetische Punkte, die nicht der Gravitation unterliegen (wichtig!) und von denen kurze Lichtblitze ausgehen (Gedankenexperiment!). Diese Lichtblitze breiten sich kugelförmig aus. Bei einer Annäherung der statischen Punkte, ergibt sich folgender Sachverhalt (siehe selbstgebasteltes Diagramm weiter unten, hier aber wieder aus dem Buch abgekupfert): Der oberste Punkt ist genau im Zentrum der kugelförmigen Lichtfront. Um so kleiner der Abstand zum Ereignishorizont wird, umso mehr verschiebt sich die Lichtfront nach vor. Am Ereignishorizont liegt der Punkt exakt an der hinteren Grenze der kugelförmigen Lichtfront. Im Schwarzen Loch befindet sich der Punkt sogar außerhalb der eigenen Lichtfront. Die Interpretation ist die, dass der Raum mit Überlichtgeschwindigkeit Richtung Singularitär gezogen wird. Der Punkt muss nacheilen, das zeigt – neben den typischen Raum-Zeit-Diagrammen (die mit den Lichtkegeln) – schön, warum ein Entkommen spätstens am Ereignishorizont unmöglich ist.

    Auch hier bewegt sich wieder nicht das Objekt mit Überlichtgeschwindigkeit sondern der Raum selbst.

    Das selbstfabrizierte, holprige Bildchen:
    https://img859.imageshack.us/img859/4100/staticpointsblackhole.jpg

    Das Buch “Kleines 1×1 der Relativitätstheorie” kann ich wie geschrieben nur empfehlen. Wer sich dafür interessiert, Oberstufenmathematik beherrscht, der ist hier richtig. Ich wurde schon oft zur SRT/ART befragt von wirklich klugen Menschen, die allerdings ohne fehlendes Studium notgedrungen bei Populärwissenschaftlichen Büchern festhingen, die mehr oder weniger gute Analogien anbieten – mehr aber nicht. Nach dem ich ihnen das Buch empfohen habe, hat sich einiges offenbar aufgeklärt zu den Fragen im Zusammenhang mit der SRT/ART. Das Buch schließt die Lücke von Populärwissenschaftlich zum Studium (Ersatz für ein Lehrbuch, bzw Fachbuch ist es natürlich trotzdem nicht). GUt genug Werbung gemacht… 😉

  84. #84 T'Pau
    20. April 2011

    Korrigendum: “Gegeben sind rein statistische Punkte”

    Gemeintist natürlich “rein _statische_ Punkte” !

  85. #85 Wolfgang Flamme
    20. April 2011

    Ich denke mal, Alexandra und volki haben genau dasselbe Verständnisproblem wie ich. Richtig?

    Ich habe zwar das Gefühl, daß die Links in Roberts Kommentar näheren Aufschluß geben könnte, aber das muß man ja erstmal nachvollziehen und langsam begreifen und so lange Frühstückspausen gibt’s gar nicht 🙂

  86. #86 Niels
    20. April 2011

    @Wolfgang Flamme
    Willkommen in der wunderbaren Welt der Relativität.

    Bob hat ein Problem mit seinem Raumschiff und fällt nun in ein schwarzes Loch.

    Für Alice, die äußere Beobachterin in sicherer Entfernung (unendlich weit weg), fällt Astronaut Bob aufgrund der Zeitdilatation immer langsamer auf das schwarze Loch zu. Die Zeitdilatation führt dann auch zu der bekannten gravitativen Rotverschiebung.
    Wenn Bob den Ereignishorizont erreicht, hört für Alice die Bewegung von Bob völlig auf. Die Zeitdilatation wird unendlich groß. Bob wird immer mehr rotverschoben, bis er nicht mehr sichtbar ist. Für Alice überschreitet er auch in unendlicher Zeit niemals den Ereignishorizont.

    Für Bob selbst sieht die Sache ganz anders aus. Er beobachtet, dass er sich immer schneller dem schwarzen Loch nähert. Er erreicht in endlicher Zeit den Ereignishorizont. Tatsächlich gibt es für ihn keine Möglichkeit, mit der er feststellen kann, dass er den Ereignishorizont überschritten hat.
    Falls Bob ein unzerstörbarer Supermann ist, beobachtet er sogar, wie er schließlich in endlicher Zeit in die Singularität selbst knallt.

    Das erscheint uns paradox, da wir es gewohnt sind, in absoluten Zeiten und Längen zu denken. Der Kern der Relativitätstheorie ist aber nun mal, dass es so etwas nicht gibt.

  87. #87 togibu
    20. April 2011

    @Nils
    bedeutet das dann aber nicht, dass für einen “unendlich weit” entfernten Beobachter nach Entstehung des BH überhaupt keine Materie mehr den Ereignishorizont durchdringt? Sondern sich nur dem Ereignishorizont immer langsamer annähert? (Mir ist schon klar, dass das aus Sicht der einstürzenden Materie nicht so ist).

  88. #88 Niels
    20. April 2011

    @togibu

    Für solche Paradoxien braucht man bei weitem nicht gleich mit schwarzen Löcher und der ART auftrumpfen.
    Da reicht auch ein schnell bewegter Stab und eine Scheune (also nur die SRT).
    Zum Beispiel
    https://itp.nat.uni-magdeburg.de/~kassner/srt/pseudoparadoxa/stab_scheune/stab_scheune.html

  89. #89 Physi
    20. April 2011

    Müsste eigentlich nicht die Unschärferelation die ‘Singularität’ beschränken auf eine minimale Größe via (delta p) (delta x) größer hquer. Ein schwarzes Loch besteht ja ursprünglich aus Teilchen, und jedes einzelne davon _kann_ nicht genau einen definierten Ort haben!

  90. #90 Bullet
    20. April 2011

    Ein schwarzes Loch besteht ja ursprünglich aus Teilchen,

    Gewagt, gewagt. Ein Kuhhaufen besteht auch ursprünglich aus Gras. Das aber ist blanke Polemik. Ich habe tatsächlich keine Ahnung, ob das, was in einem Schwarzen Loch herumflitzt, noch unsere Definition von “Materie” abbildet.

  91. #91 MartinB
    20. April 2011

    @Bullet, Physi
    So wie ich das sehe, hat Physi durchaus recht – spätestens, wenn die Singularität auf eine Plancklänge schrumpft, gibt es mächtig Ärger zwischen ART und QM. Aber solange nicht irgendwer ne Quantengravitation aus dem Hut zaubert, bleibt die Frage wohl offen.

  92. #92 Bullet
    20. April 2011

    Ha! Du konntest mich also nicht widerlegen.

  93. #93 Niels
    20. April 2011

    @MartinB
    Eine Quantengravitation, in der Singularitäten auftreten, wäre irgendwie sinnlos, oder?

    Ich glaube, du musst demnächst einfach mal einen Beitrag über schwarze Löcher schreiben.
    Nachdem du dich eh schon durch die ersten Kapitel MTW gequält hast, sollte der Rest doch ein Klacks sein… 😉

  94. #94 togibu
    21. April 2011

    @Niels (Das fehlende “e” in meinem letzten Post wird hiermit nachgeliefert)

    Das mit dem Paradoxon war mir schon klar.

    Meine Frage war aber vielmehr, ob sich aufgrund der Tatsache, dass aus unserer Sicht nach Bildung des BH keine Teilchen mehr den Ereignishorizont passieren, sich der Ereignishorizont (und damit ja das, was wir vom BH überhaupt noch beobachten können) nicht mehr vergrößert? M.a.W.: Das BH (bzw. das Volumen innerhalb der Ereignishorizonts) wächst im Volumen überhaupt nicht mehr, da der Schwarzschildradius doch von der Masse hinter dem Ereignishorizont abhängt. Oder unterliege ich einem Denk- oder Verständnisfehler?

  95. #95 Niels
    22. April 2011

    @togibu
    Äh, Schwarze Löcher wachsen doch offensichtlicherweise, oder?

    Ich glaube aber, ich verstehe dein Problem.
    Mal schaun, ob mir dazu noch etwas intelligentes einfällt.

    Hier sind aber viele kluge Leute unterwegs, die auch mehr als ich davon verstehen.
    Vielleicht kann jemand von denen noch etwas dazu sagen?

  96. #96 Bjoern
    22. April 2011

    @Niels und @togibu: Diese Frage hatte ich vor Monaten schon mal mit Andreas Müller (vom Blog “Einsteins Kosmos” auf den KosmoLogs) diskutiert, der ja Experte für SLs ist. Nachdem er die Frage erst mal verstanden hatte (was lange gedauert hat…), war die Erklärung, soweit ich mich erinnere, in etwa so: wir können zwar optisch nicht beobachten, dass Dinge vom SL verschluckt werden; trotzdem können wir messen, dass die Masse eines SLs zunimmt (da seine Gravitation halt dadurch zunimmt). Sprich: das SL wächst zwar, aber die optische Information über dieses Anwachsen kann uns halt nie erreichen. (das war das, was ich aus der Diskussion mitgenommen hatte – ich hoffe, ich stelle Andreas hier gerade nicht falsch dar…)

  97. #97 HAR1
    22. April 2011

    Ach, der Troll aus Thailand mal wieder… Tschüß!

  98. #98 MartinB
    22. April 2011

    @Niels
    Nee, die wirren Schwarzen-Loch-Metriken mit Umkehr von Raum- und Zeitkoordinaten hab ich noch nicht so richtig gecheckt, deswegen lass ich das mit dem Schreiben lieber.
    Was die Singularitäten angeht – wer weiß schon, was die QG bringt; nachher stellt sich heraus, dass alles aus Singularitäten besteht…

    Bezüglich des Wachsens: Da die reinstrzende Masse ja die Gesamtmasse erhöht, wächst auch der Schwarzschildradius mit. ich könnte mir vorstellen (ohne das beweisen zu können), dass man auf die Weise das Schwarze Loch zum Wachsen bekommt. In dem einfachen “Objekt stürzt ins SL”-Modell wird ja die Masse des reinfallenden Objekts als sehr klein angenommen und nicht berücksichtigt, dass es selbst die Raumzeit auch krümmen muss. So funktioniert es doch sicher auch, wenn zwei SL verschmelzen – sobald sie sich nahe genug kommen, “verschmelzen” die Ereignishorizonte zu einem größeren.
    Aber wie gesagt, das ist nur Intuition, nicht sauber gerechnet, mag also falsch sein.

  99. #99 Niels
    22. April 2011

    @Bjoern
    Wäre super, wenn du diese Diskussion mal verlinken könntest. Oder weißt du noch, unter welchem Blogbeitrag diskutiert wurde?
    Spontan hab ich bei “Einsteins Kosmos” nix gefunden.

  100. #100 Bjoern
    22. April 2011

    @Niels: Die Diskussion war auch nicht auf Andreas’ Blog. Leider habe ich keine Ahnung mehr, wo sie denn nun eigentlich sonst war… (evtl. hier auf Florians Blog – aber frag’ mich nicht, in welchem Beitrag)

  101. #101 Niels
    22. April 2011

    @MartinB
    Diese Intuition klingt aber zumindest schon mal plausibel.

    Schwarze Löcher stehen bei mir auf der Liste von Themen, die ich mir mal richtig anschauen will. Die Liste ist aber irgendwie schon ziemlich lang und seltsamerweise bekomm ich das nötige Jahr Forschungsurlaub nicht genehmigt. 😉

    @Bjoern
    Schade.

  102. #102 MartinB
    22. April 2011

    @Niels
    Zum verstehen ist nichts besser als zu versuchen, es anderen zu erklären. Falls du mal einen Gasteintrag bei mir schreiben willst… (Bei Florian bekommst du allerdings mehr klicks.)

  103. #103 Wolfgang Flamme
    22. April 2011

    @Bjoern

    Diese Erklärung finde ich irgendwie total unbefriedigend. Wenn ein außenstehender Beobachter seit 14 Mrd Jahren oder so keine Materie/Energie beim Übergang in den Ereignishorizont eines SLs hat beobachten können (sondern nur, wie sie diesem sehr, sehr nah gekommen ist), dann kann er auch keine Wirkungen von Materie sehen oder messen, die den Ereignishorizont überschritten hat – weil es eben in seiner Raumzeit noch nicht geschehen ist … und erst in unendlich ferner Zukunft geschehen wird.

    Das einzig positive an dieser Diskussion ist für mich bisher, daß ich nicht der einzige bin, der diese doch recht einfache und naheliegende Frage seit langem mit sich herumschleppt, ohne daß er sie beantwortet bekommen hat.

  104. #104 Ben
    25. April 2011

    Ich hab das selbe Problem wie Wolfgang Flamme, togibu usw.

    “”war die Erklärung, soweit ich mich erinnere, in etwa so: wir können zwar optisch nicht beobachten, dass Dinge vom SL verschluckt werden; trotzdem können wir messen, dass die Masse eines SLs zunimmt””

    Wenn die Zeitdilatation für den äußeren Zuschauer wirklich unendlich wird, dann kann man doch nicht einfach als Lösung behaupten: Das Objekt fällt doch ins Loch, nur die optische Information erreicht den Zuschauer nicht.
    Bei unendlicher Zeitdilatation bewegen sich das Objekt doch nicht.

    Kann da jemand von den Experten das nochmal ausführlich erklären?

  105. #105 perk
    25. April 2011

    ich war bisher immer ein fan von andreas müllers astrolexikon.. aber cauchy-fläche und cauchy horizont als begriffe für das gleiche objekt zu benutzen ist extrem irreführend

    eine cauchyfläche ist eine achronale hyperfläche die von jeder kausalen kurve der raumzeit exakt einmal geschnitten wird

    eine partielle cauchy fläche wird von jeder kausalen kurve der raumzeit höchstens einmal geschnitten

    aber beide haben die eigenschaft dass sie für einen gewissen bereich in ihrer zukunft die physik vollständig bestimmen, da in diesen bereich der raumzeit (future cauchy development D+) nur kausale kurven eindringen die in ihrer vergangenheit die (partielle) cauchyfläche passiert haben

    es gibt also auf einen beobachter innerhalb der D+ keinen einfluss aus der nicht auch schon auf die cauchy fläche wirkte, sie reicht also im sinne der lösungsbedingungen für differentialgleichungen als anfangsbedingung aus um die physik in D+ zu beschreiben

    so nun zum horizont: der (zukünftige) cauchy horizont ist die grenze zwischen D+ und der umgebenden raumzeit, also die grenze zwischen dem raum der vollständig durch die daten auf der cauchyfläche bestimmt wird und dem raum indem auch andere einflüsse wirken

    Aber es handelt sich um eine Fläche, die ein wenig wie eine halbdurchlässige Membran funktioniert: man kann sie nur einmal durchqueren und dann bleibt man drinnen.

    das macht bezüglich einer (partiellen) cauchy fläche auch keinen sinn.. da man ja in der zukunft dessen ist.. da gibt es kein drinnen oder draußen sondern eher ein zukünftig und vergangen

    du würdest ja auch nicht sagen dass das “passieren” des jahres 2000 dazu geführt hat dass wir jetzt in der zeit nach 2000 eingesperrt (drinnen) sind

    in kurz: das passieren einer cauchyfläche sagt nichts über die erreichbarkeit anderer raumbereiche aus

    die bildliche vorstellung die du da beschreibst klingt eher nach einer trapped surface (also einer bei der beide zukunftsgerichteten lichtartigen geodätenscharen konvergieren und es somit nur noch nach innen weiter geht (solange man günstige energiebedingungen und kausalitätsbedingungen anlegt kann man der region dann auch nicht mehr entkommen))

    als umfassendste referenz für das thema würd ich global lorentzian geometry empfehlen

  106. #106 perk
    25. April 2011

    achso vor allem für niels und martinb noch ein lesetipp: https://www.springerlink.com/content/vj5013u0136t5273/ das hat bei mir für einige mächtige “oha”-momente gesorgt.. die vorstellungen die man so aus der populären beschäftigung mit schwarzen löchern gewinnt sind ziemlich irreführend und die wahl der extension hinter den ereignishorizont ist noch nichtmal im schwarzschild-fall physikalisch wohlbegründet (so wie ich es verstanden habe muss die erweiterung singulär sein aber es kommen noch verschiedene singularitäten in betracht)

  107. #107 Ben
    25. April 2011

    @perk
    Kannst du auch was zu der noch offenen Frage sagen?

  108. #108 Christian 2
    26. April 2011

    Interessant, aber doch sehr spekulativ. Man kommt erstmal ja nur hinein, wenn das schwarze Loch ausreichend Masse hat und sich wie das unserer Galaxie ruhig verhält. Ist das schwarze Loch klein so kommt man nicht besonders nah heran, bevor man bereits von der Gravitation zerquetscht wird.

    Wird man nicht in der Nähe des Ereignishorizontes irgendwann eh zerquetscht? Ich wüsste nicht, wie man das überleben können sollte. Nicht in Form von fester Materie.

    Wie brutal schwarze Löcher mit Materie interagieren sieht man ja z.B. an den ausgestoßenen Jets. Etwas zerstörerischeres gibt es kaum.
    Da über mögliches Leben in dieser Hölle zu spekulieren, ist äußerst gewagt.

    Hinzu kommt z.B. das Problem, das die Zeit immer langsamer abläuft und irgendwann zum Stillstand kommt usw.

    Woher will man etwas über eine Welt erfahren, die der Wissenschaft aufgrund der vorhandenen Gesetzmäßigkeiten verschlossen bleibt?

    Man kann dort eigentlich keine fundierte Wissenschaft mehr betreiben. Das ist wie der Versuch, anhand von theoretischen Berechnungen eine Welt hinter dem Universum herzuleiten.
    So etwas ist zwar spannend zu lesen, aber auch nicht unbedingt gehaltvoller als ein Science Fictionroman.

  109. #109 perk
    26. April 2011

    @ ben ich vermute mal es geht um eine anschaulichere erklärung warum das hier:

    Wenn die Zeitdilatation für den äußeren Zuschauer wirklich unendlich wird, dann kann man doch nicht einfach als Lösung behaupten: Das Objekt fällt doch ins Loch, nur die optische Information erreicht den Zuschauer nicht.

    tatsächlich funktioniert obwohls erstmal komisch klingt

    nun ich kann noch eine anschauung anbieten die eventuell hilft: das integral über die eigenzeit entlang einer weltlinie eines beobachters der den EH erreicht ist endlich.. er kommt also wirklich dort an..

    die relativen richtungen von raum und zeit ändern sich aber kontinuierlich auf diesem weg bis auf dem EH die lichtartige richtung für radial nach außen ausgesandte photonen genau parallel zum EH ist (sie können ihn also nicht verlassen)

    wenn man sich das so geometrisch vorstellt gerät man nicht in den konflikt die zeitdilatationen der beiden beobachter zu vergleichen: es gibt einfach kein photon mit unendlicher rotverschiebung das den beobachter im unendlichen erreicht.. sondern das photon ist aufgrund der am EH vorliegendenen richtungen dazu gezwungen im EH zu bleiben (bzw bei kleinen störungen (minimaler massezuwachs des bh) nach innen zu fallen)

    dieser richtungswechsel stellt für unseren hineinfallenden beobachter allerdings kein problem dar, da er ja eh weiter in richtung zentrum will und es ihn nicht stört wenn die nach außen zeigenden richtungen plötzlich raumartig und damit unerreichbar werden (und einige raumartige richtungen nach innen zeitartig und damit erreichbar)

  110. #110 Niels
    26. April 2011

    @perk
    Danke für die Literaturtipps. Der Link für “global lorentzian geometry” funktioniert nicht.
    Ist das hier gemeint? https://www.amazon.de/Global-Lorentzian-Geometry-Applied-Mathematics/dp/0824793242
    Sieht wirklich umfassend aus.
    Der Review aus der Fachzeitschrift hat auch 148 Seiten. Uff…

    Möchtest du nicht mal bei MartinB einen Gastbeitrag schreiben? 😉

  111. #111 perk
    26. April 2011

    hat martin mir schon angeboten.. aber ich bin noch nicht weit genug.. 😉

    Ist das hier gemeint? https://www.amazon.de/Global-Lorentzian-Geometry-Applied-Mathematics/dp/0824793242

    jupp das jenige meine ich.. ich hab zum glück das unibibo exemplar unbefristet geliehen bekommen.. ok n bissel mehr druck würde meinem vorankommen auch besser tun 😉

  112. #112 MartinS
    27. April 2011

    Es passt vielleicht nicht so ganz zu ‘Leben in SLs’, aber es macht es vielleicht etwas ‘möglicher’! ⇒hier wird gezeigt, dass Bakterien bis zu >400.000 G überstehen können. (Eine Nachrichtenmeldung dazu habe ich ⇒hier gefunden.) Ob die Erkenntnisse eine Relevanz für SLs haben oder nur die Panspermie untermauern, überlasse ich eurer Beurteilung.

  113. #113 Christian 2
    27. April 2011

    Man sollte erstmal die Außerirdischen finden, bevor man anfängt über Leben in zerstörerischen Ungeheuern zu spekulieren. Ich frage mich immer noch wie man vom Beobachter sprechen kann wo doch klar ist, das komplexes Leben niemals in den Genuss der Beobachtung im Schwarzschildradius käme?
    Man stirbt dort. Oder habe ich was verpasst?

    Man spricht immer vom Beobachter, und dem Erlebten. Dabei ist das Blödsinn. Ein Beobachter kann sich z.B. nicht mit 99% der Lichtgeschwindigkeit fortbewegen, ohne das er auf der Stelle aufhören würde zu existieren. Für mich persönlich ist diese Anschauung für Unwissende einfacher Nonsens. Einfach, da dieses physikalisch nicht funktioniert.

    Unter welchen Extremen welches Leben existieren könnte- Das ist eine Frage, die wir bislang nur von der Erde beantworten können. Und die Erde ist im Vergleich zu solchen Orten ein Paradies.

    Steckt lieber eure theoretische Energie in die Erfindung eines Warpantriebs. Langfristig gesehen haben wir da mehr von.

  114. #114 Bjoern
    27. April 2011

    @Christian2:

    …wo doch klar ist, das komplexes Leben niemals in den Genuss der Beobachtung im Schwarzschildradius käme?
    Man stirbt dort. Oder habe ich was verpasst?

    Warum sollte man dort automatisch sterben? Spielst du auf die “Spaghettification” durch die Gezeitenkräfte an? Falls ja: die Gezeitenkräfte sind bei sehr großen schwarzen Löchern (z. B. solche im Zentrum der Milchstraße) vernachlässigbar.

    Ein Beobachter kann sich z.B. nicht mit 99% der Lichtgeschwindigkeit fortbewegen, ohne das er auf der Stelle aufhören würde zu existieren.

    Auch hier: warum? Meinst du die kosmische Strahlung? Die könnte man doch mit entsprechenden technischen Mitteln abschirmen?

  115. #115 Niels
    27. April 2011

    @perk
    Warum beschäftigst du dich eigentlich momentan damit? Beruflich oder zum Spaß?
    Was ist das “Ziel”?
    Anscheinend gehst du ziemlich in die Tiefe.

  116. #116 Christian 2
    27. April 2011

    @ Bjoern: Ich habe mal gelesen, das große schwarze Löcher jeden Tag etwa die Materie eines Planeten verschlucken. Wobei das von der Aktivität bzw. der umgebenden Materie abhängt. Desto weniger, desto inaktiver.

    Ein Planet mag nicht viel sein. Ich bezweifel aber, das man den Raumflug in ein schwarzes Loch überleben würde. Man kommt bei den Großen nur weiter hinein. Die Anziehungskraft im Zentrum ist ja weiterhin enorm. Vor allem bei den Massen, von denen wir sprechen (Mio. Sonnenmassen).

    Abschirmen von harter Gammastrahlung? Ich kenne keinen Weg, wie man derartiges abschirmen können sollte. Hinzu kommt, das jede Kollision mit kleineren millimetergroßen Gesteinsbrocken wohl tödlich enden würde. Die Wucht des Aufschlags auf ein Objekt mit Lichtgeschwindigkeit wäre enorm, und mit heutigen technischen Mitteln nicht abwendbar.
    Hinzu kommt die Beschleunigung der Masse. Sowas ist nicht mehr mit einem Düsenflugzeug vergleichbar. G- Kräfte die hier auftreten könnten, sind für feste Massen aller Art ein echtes Problem.

  117. #117 MartinB
    27. April 2011

    @Christian
    Beim Sturz ine in schwarzes Loch treten keine G-Kräfte auf – man fällt immer schwerelos (weil es keine Schwerkraft gibt…).
    Gezeitenkräfte sind das Problem, die sind bei großen Schwarzen Löchern aber klein.
    https://www.scienceblogs.de/hier-wohnen-drachen/2011/02/wie-man-die-raumzeit-krummt-teil-v.php

  118. #118 Christian 2
    27. April 2011

    G- Kräfte und harte Gammastrahlung bezogen sich auf die Beschleunigung eines Raumschiffs auf nahezu LG. Die Strahlung dort in der Nähe des Galaxiezentrums ist aber sicher auch nicht zu verachten. Wenn man sich überlegt was dort alles herumfliegt, ist ein Aufenthalt dort schon 100%tig tödlich.

    Das große Massen keine großen Gezeitenkräfte verursachen, nehme ich dir nicht ab. Dieser Effekt mag aufgrund der schieren Größe des schwarzen Lochs und somit einem großen Abstand zum Zentrum schwächer sein. Das bedeutet aber nicht, das man bei einem Flug in ein solches Monster nicht irgendwann zerquetscht würde.

  119. #119 Bjoern
    27. April 2011

    @Christian2:

    Ich bezweifel aber, das man den Raumflug in ein schwarzes Loch überleben würde. Man kommt bei den Großen nur weiter hinein. Die Anziehungskraft im Zentrum ist ja weiterhin enorm.

    Äh, reden wir jetzt vom Überschreiten des Ereignishorizonts (der bei großen SL sehr weit vom Zentrum entfernt ist!), oder von einer Annäherung an die eigentliche Singularität im Zentrum?

    Abschirmen von harter Gammastrahlung? Ich kenne keinen Weg, wie man derartiges abschirmen können sollte. Hinzu kommt, das jede Kollision mit kleineren millimetergroßen Gesteinsbrocken wohl tödlich enden würde. Die Wucht des Aufschlags auf ein Objekt mit Lichtgeschwindigkeit wäre enorm, und mit heutigen technischen Mitteln nicht abwendbar.

    Man nehme z. B. einen größeren Asteroiden und lasse den vor sich her fliegen. Technisch natürlich sehr schwierig, aber eindeutig prinzipiell machbar.

    Das große Massen keine großen Gezeitenkräfte verursachen, nehme ich dir nicht ab. Dieser Effekt mag aufgrund der schieren Größe des schwarzen Lochs und somit einem großen Abstand zum Zentrum schwächer sein. Das bedeutet aber nicht, das man bei einem Flug in ein solches Monster nicht irgendwann zerquetscht würde.

    Wenn du’s nicht glaubst, dann rechne es halt selber durch! Wenn du nicht weisst, wie: die nötige Formel findest du im entsprechenden Wikipedia-Artikel. (ich habe gerade erst Herrn E.K. seitenweise was vorgerechnet, ich hab’ echt keine Lust, das auch noch mit dir hier anzufangen…)
    https://de.wikipedia.org/wiki/Gezeitenkraft

  120. #120 SCHWAR_A
    27. April 2011

    Gibt es ‘Leben’ in schwarzen Löchern?

    Beweis:

    Die Gesamtmasse M des beobachtbaren Universums ist etwa 8.5·10^52kg.
    Daraus ergibt sich der zugehörige Schwarzschild-Radius zu 2GM/c^2 = 1.3·10^26m, also dem Radius des beobachbaren Universums.

    Der Ereignishorizont eines nicht rotierenden SL ist durch den Schwarzschild-Radius beschrieben. Wir ‘leben’. Also gibt es ‘Leben’ im Schwarzen Loch! q.e.d. 😉

  121. #121 MartinB
    27. April 2011

    @Christian2
    Na klar, irgendwann wird man zerquetscht – bei einem hinreichend großen Schwarzen Loch aber erst weit innerhalb des Ereignishorizonts.
    Was das auf nahezu c beschleunigende Raumschiff damit zu tun hat, ist mir nicht klar – aber ein Jahr bei einem g bringt einen auf nahe Lichtgeschwindigkeit – man bräuchte natürlich ne gute Abschirmung gegen Atome etc.

  122. #122 Wolfgang Flamme
    27. April 2011

    @perk

    Ich habe da immer noch ein gewaltiges Problem mit dieser Erklärung. Es wird wohl an zwei Dingen liegen: a) zuwenig fanatisches Interesse bzw Ehrgeiz und b) ‘Altlast Hessenabitur’. 🙂

    Uns hat man in Physik halt grundsätzlich eingehämmert “Uhren in einem Gravitationsfeld gehen aus Sicht ‘eines außenstehenden Beobachters’ langsamer. Nicht nur, daß das aufgrund eines Problems mit endlicher und relativ-konstanter Kommunikationsgeschwindigkeit bloß so aussieht, sondern daß es vielmehr gerade deshalb so IST.

    Insofern habe ich natürlich kein Problem zu schließen, daß ein auf den Ereighnishorizont zustürzender und sogar durch ihn hindurchstürzender Beobachter das tatsächlich in (s)einer endlichen Zeit erleben wird. Aber ich bekomme Kopfschmerzen bei der Vorstellung, dieser Übergang könnte in meiner endlichen endlichen Zukunft geschehen und beobachtbar sein.

     Und wenn ich mir zuletzt sogar vorzustellen versuche, daß mich von dem Durchsturz ‘lediglich’ keine Photonen erreichen können, aber die Information über den dadurch bewirkten Massezuwachs schon, dann riskiere ich schon leichte Hirnblutungen.

    Also ich muß diese Pseudo- ART wohl erstmal komplett vergessen und dann von Grund auf neu lernen. Keine ganz leichte Aufgabe.

  123. #123 Niels
    28. April 2011

    @SCHWAR_A

    So einfach ist das alles auch nicht.

    Deine 1.3•10^26 Meter sind ungefähr 13,74 Milliarden Lichtjahre. Das ist etwa Alter des Universums mal Lichtgeschwindigkeit. Nur hat das mit der tatsächlichen Größe des beobachtbaren Universums nichts zu tun.
    https://www.scienceblogs.de/hier-wohnen-drachen/2010/09/wie-gross-ist-das-beobachtbare-universum.php
    Der Radius des beobachtbaren Universums ist also in Wirklichkeit 48 Milliarden Lichtjahre.

    Es gibt aber tatsächlich einen sogenannten kosmologischen Ereignishorizont. Dieser hat aber nichts mit dem Radius des beobachtbaren Universums zu tun.
    Der kosmologische Ereignishorizont ist ungefähr etwa 15,6 Milliarden Lichtjahre entfernt.
    Nicht zuletzt sind beide Größen aber zeitabhängig, obwohl sich die Gesamtmasse des Universums im Wesentlichen nicht verändert.
    Was ist bei dir überhaupt die Gesamtmasse M? Da ist die dunkle Energie wahrscheinlich nicht dabei, obwohl sie 73% der Energiedichte des Universums liefert.
    Dürfte also nicht vernachlässigbar sein, oder?
    Wenn man diese Größen wirklich berechnen will, muss man an die Friedmann-Gleichungen ran. Da gibts keine Abkürzung.

  124. #124 Niels
    28. April 2011

    Dein berechneter Schwarzschild-Radius liefert nur völlig zufällig das selbe Ergebnis wie die Rechnung heutiges Universumsalter mal Lichtgeschwindigkeit.
    Es gibt aber wirklich Überlegungen, dass sich in schwarzen Löchern Universen befinden und auch wir in einem schwarzen Loch leben. Das ist aber nur eine wilde Spekulation und auch deutlich komplizierter, als du es dir anscheinend vorstellst. Du muss man Wurmlöcher bemühen usw.
    Das ist wohl die maßgebliche Veröffentlichung dazu:
    https://www.sciencedirect.com/science?_ob=ArticleURL&_udi=B6TVN-4YK7J05-3&_user=10&_coverDate=04%2F12%2F2010&_rdoc=1&_fmt=high&_orig=gateway&_origin=gateway&_sort=d&_docanchor=&view=c&_acct=C000050221&_version=1&_urlVersion=0&_userid=10&md5=2258c347977bebab365dd85bbfcce6ce&searchtype=a

    These results suggest that observed astrophysical black holes may be Einstein-Rosen bridges, each with a new universe inside that formed simultaneously with the black hole. Accordingly, our own Universe may be the interior of a black hole existing inside another universe.

    Hier eine populärwissenschaftliche Meldung:
    https://www.sciencedaily.com/releases/2010/04/100406172648.htm
    Wie gesagt, dass ist ne nette Idee, wird aber keineswegs von der Mehrheit vertreten.

  125. #125 perk
    28. April 2011

    Uns hat man in Physik halt grundsätzlich eingehämmert “Uhren in einem Gravitationsfeld gehen aus Sicht ‘eines außenstehenden Beobachters’ langsamer.

    das tun die uhren des hereinfallenden beobachters auch, aber das gravitationsfeld (und damit die zeitdilation der mitgeführten uhr) ist endlich am EH

    Aber ich bekomme Kopfschmerzen bei der Vorstellung, dieser Übergang könnte in meiner endlichen endlichen Zukunft geschehen und beobachtbar sein.

    der übergang ist von allen beobachtern wahrnehmbar, die im zukunftslichtkegel des übergangs liegen (und nur denen).. die sind leider alle innerhalb des schwarzen lochs 😉

    Und wenn ich mir zuletzt sogar vorzustellen versuche, daß mich von dem Durchsturz ‘lediglich’ keine Photonen erreichen können, aber die Information über den dadurch bewirkten Massezuwachs schon, dann riskiere ich schon leichte Hirnblutungen.

    es gibt keine direkten informationen über den übergang (und damit den massenzuwachs) im außenraum.. das gravitationsfeld des systems SL + hineinstürzender körper ändert sich kontinuierlich und ist deshalb infinitesimal vor dem überqueren des EH nicht anders als danach

    indirekt könnte man den massenzuwachs später mit einem erneuten probekörper überprüfen von dem man n paar millimeter eher schon keine photonen mehr empfängt

    hilft das irgendetwas?

  126. #126 SCHWAR_A
    28. April 2011

    @Niels:

    Wenn die Urknall-Hypothese im SL ihre Gültigkeit beibehält, hast Du natürlich Recht.
    Wenn aber nicht, dann erübrigt sich ebenfalls die Frage nach der Dunklen Energie in diesem Zusammenhang…
    Die kritische Massendichte ρ_c wird ja m.W. nach aus den Friedmann-Gleichungen gewonnen. Die daraus errechnete Gesamtmasse sollte also eigentlich schon alles umfassen, oder?

    Zu sagen, daß das “völlig zufällig” so ist: na, na, soweit würde ich mich lieber nicht aus dem Fenster lehnen…

  127. #127 Bjoern
    28. April 2011

    @perk:

    das gravitationsfeld des systems SL + hineinstürzender körper ändert sich kontinuierlich und ist deshalb infinitesimal vor dem überqueren des EH nicht anders als danach

    Das Gravitationsfeld ist doch vor dem Hineinstürzen nicht zentralsymmetrisch; wenn der hineinstürzende Körper “innen” angekommen ist, sollte es aber doch wieder zentralsymmetrisch werden?

  128. #128 Wolfgang Flamme
    28. April 2011

    @perk

    das hilft, indem es durchaus in die Richtung geht, die Roberts Lit.-Verweis oben bereits angedeutet hat. Demnach wäre die gravitative Zeitdilatation am Ereignishorizont (Grenzfall) also nicht unendlich groß? (ich kann da nur leider schon die als gegeben vorausgesetzte Einstiegsformel nicht nachvollziehen, soweit bin ich eben nicht eingetaucht).

    Das widerspricht alledings genau meinem (angestaubten) relativistischem Schulwissen, wonach der Faktor für die gravitative Zeitdilatation sqrt(1 – 2GM/R/c²) beträgt, wobei im Fall eines ‘trivialen’ schwarzen Lochs am Ereignishorizont R (=R_schwarzschild) die gravitative Zeitdilatation eben grenzwertig würde; wg. 2GM/R_schwarzschild/c²=1

    Und ich denke irgendwo auch, wenn es so trivial zu zeigen wäre, daß die gravitative Zeitdilatation am Ereignishorizont endlich ist, dann würde man das ja wahrscheilich in jedem Schulphysikbuch oder Astronomieschmöker und jedem Wissenschaftsforum hergeleitet bekommen – zumal dieses Paradoxon ja doch eine recht verbreitete Kopfnuß zu sein scheint.

  129. #129 perk
    28. April 2011

    Das Gravitationsfeld ist doch vor dem Hineinstürzen nicht zentralsymmetrisch; wenn der hineinstürzende Körper “innen” angekommen ist, sollte es aber doch wieder zentralsymmetrisch werden?

    es gibt keinen grund warum es zentralsymmetrisch sein sollte, die masse ist nach dem passieren des eh trotzdem erstmal noch kurz unter der oberfläche.. die raumzeit ist also auch entsprechend asymmetrisch verformt

    wird halt kartoffelig: https://de.arxiv.org/pdf/gr-qc/0012079v2

    @ wolfgang flamme: in der schwarzschild-metrik wird das eigenzeitelement ds am ereignishorizont singulär, das ist aber nur ein problem der schlechten koordinaten

    es gibt keine absoluten zeitdilatationen.. sondern immer nur relative und die bekommt man durch integrationen entlang von weltlinien heraus.. da es aber keine kausale kurve gibt, die vom EH zu nem außenstehenden beobachter geht, ist kein wert für eine zeitdilatation definiert..

  130. #130 Bjoern
    28. April 2011

    @perk:

    es gibt keine absoluten zeitdilatationen.. sondern immer nur relative und die bekommt man durch integrationen entlang von weltlinien heraus..

    Ich dachte immer, die relative Zeitdilation für zwei Beobachter wäre einfach das Verhältnis der Komponenten g_00 des metrischen Tensors an den Orten/Zeiten der beiden Beobachter…?

  131. #131 MartinB
    28. April 2011

    @perk
    Jetzt bin ich verwirrt – widerspricht das nicht dem “no-hair”-Theorem, wenn man von Außen eine “kartoffeligkeit” sehen kann?

  132. #132 perk
    28. April 2011

    wikipedia sagt dazu:

    More generally, every unstable black hole decays rapidly to a stable black hole; and (modulo quantum fluctuations) stable black holes can be completely described at any moment in time by these eleven numbers:
    mass-energy M,
    linear momentum P (three components),
    angular momentum J (three components),
    position X (three components),
    electric charge Q.

    ich glaube das no-hair gilt nur für zeitunabhängige schwarze löcher und bei dem verschlucken von masse ist die zeitunabhängigkeit nicht mehr gegeben

    hmmm

  133. #133 MartinB
    28. April 2011

    @perk
    Ja, das klingt vernünftig, hätte ich auch selbst drauf kommen können…

  134. #134 Niels
    28. April 2011

    @ perk
    Das hier hat doch ziemlich viel mit dem Thema zu tun, oder?
    https://xxx.uni-augsburg.de/PS_cache/gr-qc/pdf/0306/0306115v1.pdf
    Anscheinend muss man statt dem Ereignishorizont einen anderen Horizont betrachten, den sogenannten dynamischen Horizont.
    Hab das aber nicht richtig angeschaut.

    @ Wolfgang Flamme
    In Schwarzschild-Koordinaten tritt am Schwarzschild-Radius, also dem Ereignishorizont, eine Singularität auf. Das siehst du schon richtig.
    Das ist allerdings keine wirkliche physikalische Singularität sondern eine sogenannte Koordinatensingularität. Diese Singularität heißt so, weil sie nicht “echt” ist, sondern bei geeigneter Koordinatenwahl verschwindet. Sie hat also keine tatsächliche Bedeutung wie die echte Singularitäten im Zentrum des Schwarzen Loches. Die Koordinatensingularität zeigt nur, dass Schwarzschild-Koordinaten für die genaue Beschreibung der Situation am Ereingishorizont einfach eine ungeeignete Wahl sind.
    In unserem Fall heißen geeignete Koordinaten zum Beispiel Kruskal-Szekeres-Koordinaten. In diesem Koordinatensystem tritt am Ereignishorizont keine Singularität auf, da wird also nichts unendlich.
    Das ist alles aber ziemlich technisch und kompliziert und mein ganzer Text läuft leider darauf hinaus, dass du das letztlich einfach glauben musst.

    @ SCHWAR_A
    Wenn die Gesamtmasse des Universums aus der kritischen Dichte berechnet wurde ist die dunkle Energie tatsächlich schon mit drin. Wo hast du diese Zahl eigentlich her?

    Zu sagen, daß das “völlig zufällig” so ist: na, na, soweit würde ich mich lieber nicht aus dem Fenster lehnen…

    Universumsalter mal Lichtgeschwindigkeit hat aber doch überhaupt keine physikalische Bedeutung! Das ist eine völlig willkürliche Rechnung.
    Warum soll jetzt eine tiefere Bedeutung dahinterstecken, wenn das Ergebnis dieser willkürlichen Rechnung ungefähr so ähnlich ist wie der Schwartzschildradius der Gesamtmasse des Universums?
    Was ist an diesem Vergleich sinnvoller als am Vergleich Schwartzschildradius zu Sonnensystemradius geteilt durch Protonenradius?

  135. #135 SCHWAR_A
    28. April 2011

    @Niels:

    Wo hast du diese Zahl eigentlich her?

    Ich denke, Du meinst die 8.5·10^52kg.
    Die stammen von hier, letzter Paragraph.
    Außerdem kann die kritische Dichte von 9.7·10^-27kg/m^3 mit dem Volumen aus dem Radius von 1.3·10^26m multipliziert werden und man erhält in etwa dieselbe Masse.

    Ich rechne gar nicht mit dem ‘Universumsalter’, sondern nur mit einem Kugelradius, dem Schwarzschild-Radius eines SL. Die Deutung als ‘Alter’ ist hier nur zulässig, wenn damit die Laufzeit von Licht für diese Strecke gemeint ist – und dann ist sie physikalisch bedeutend: wir bewegen uns ja hier im hypothetischen ‘nicht-expansiven-SL’.

    Was mich zuerst so stutzig gemacht hat: ich ging bisher immer davon aus, daß SL sehr kompakt und dicht sein müssen. Das scheint aber tatsächlich gar nicht notwendig zu sein.

  136. #136 Niels
    28. April 2011

    @SCHWAR_A

    Außerdem kann die kritische Dichte von 9.7·10^-27kg/m^3 mit dem Volumen aus dem Radius von 1.3·10^26m multipliziert werden und man erhält in etwa dieselbe Masse.

    Man erhält 2*10^52 kg, nicht 8.5·10^52kg. Also schon mal um den Faktor 4 falsch.
    Außerdem ist 1.3·10^26 Meter (13,74 Milliarden Lichtjahre) nun einmal einfach nicht der Radius des beobachtbaren Universums.

    Ich rechne gar nicht mit dem ‘Universumsalter’, sondern nur mit einem Kugelradius, dem Schwarzschild-Radius eines SL.

    Mit welcher physikalischen Größe vergleichst du den von dir berechneten Schwarzschild-Radius 2GM/c^2 = 1.3·10^26 Meter ?

  137. #137 SCHWAR_A
    28. April 2011

    @Niels:

    4π/3=4.18879…

  138. #138 SCHWAR_A
    28. April 2011

    @Niels:

    Mit welcher physikalischen Größe vergleichst du den von dir berechneten Schwarzschild-Radius…

    Mit dem Radius eines Ω_M=0 Λ=0 Universums, also mit dem Hubble-Radius, also dem Abstand zu uns, der dem Licht von dort eine unendliche Rotverschiebung zufügt.

    Ich ahne schon, Du sagst jetzt bestimmt: “Aha, also doch Expansion! –> Widerspruch!”.
    Das nehme ich mal so hin – das ganze ist ja auch nur hypothetisch – und darin könnte die Rotverschiebung ja auch anders als durch Expansion definiert sein…

    Gibt es eigentlich Aussagen zur Dichte in ‘echten’ SL?

  139. #139 Niels
    28. April 2011

    @SCHWAR_A
    Der Hubble-Radius fügt dem Licht aber keine unendliche Rotverschiebung zu.
    Wie kommst du darauf?
    Weißt du, dass der Huble-Radius zeitabhängig ist? Vom Hubble-Radius zu welchem Universumsalter sprichst du also?
    Warum soll der Hubble-Radius dem Radius des beobachtbaren Universums eines Ω_M=0 Λ=0 Universums entsprechen?
    Ist dir klar, dass der Hubble-Radius eines Universums ohne Materie und dunkle Energie sich völlig anders entwickeln würde als der Hubble-Radius unseres Universums und dass zum Beispiel auch der Hubble-Radius zum heutigen Universumsalter ein völliger anderer wäre?

    Gibts eigentlich außer dir noch irgend jemand, der diese Idee vertritt?

  140. #140 SCHWAR_A
    28. April 2011

    @Niels:
    Die Idee kam mir spontan beim Rumrechnen mit dem Schwarzschildradius, mehr nicht… Ich sagte ja auch ständig “hypothetisch” und “nicht-expandierend”…

    Was mich einfach so stutzig gemacht hat: ich ging bisher immer davon aus, daß SL sehr kompakt und dicht sein müssen. Das scheint aber tatsächlich gar nicht notwendig zu sein. Im Gegenteil: die mittl. Dichte nimmt sogar quadratisch mit ihrer Masse bzw. ihrer Größe ab. Aber kann man das bis ins Unendliche ausdehnen? Ich denke nicht. Das größte bisher entdeckte SL hat einen Radius von etwa 5·10^13m und eine mittl. Dichte von nur 56g/m^3.

    Wenn es aufgrund dieses Verhaltens eine Größenbeschränkung für SL gibt, müßte das einen Grund haben. Wird zB. die Gravitation um das Zentrum eines SL in einem stärkeren Maße schwächer als mit 1/r^2, z.B. dadurch, daß sie ebenfalls der Gravitation unterliegt und somit weniger Kraft ausüben kann?

    Wie würde die Rotverschiebung von Licht verlaufen, das im Zentrum eines SL startet?
    Wie verläuft die Blauverschiebung von Licht, das am Ereignishorizont startet und in Richtung Zentrum läuft?
    Ist die Dichte im SL überall gleich?
    Im Zentrum herrscht zumindest Schwerelosigkeit…

  141. #141 SCHWAR_A
    28. April 2011

    @Niels:
    Die Idee kam mir spontan beim Rumrechnen mit dem Schwarzschildradius, mehr nicht… Ich sagte ja auch ständig “hypothetisch” und “nicht-expandierend”…

    Was mich einfach so stutzig gemacht hat: ich ging bisher immer davon aus, daß ein SL sehr kompakt und dicht sein muß. Das scheint aber tatsächlich gar nicht notwendig zu sein. Im Gegenteil: seine mittl. Dichte nimmt sogar quadratisch mit seiner Masse bzw. Größe ab. Aber kann man das bis ins Unendliche ausdehnen? Ich denke nicht. Das größte bisher entdeckte SL hat einen Radius von etwa 5·10^13m und eine mittl. Dichte von nur 56g/m^3.

    Wenn es aufgrund dieses Verhaltens eine Größenbeschränkung für SL gibt, müßte das einen Grund haben. Wird zB. die Gravitation innerhalb eines SL in einem stärkeren Maße schwächer als mit 1/r^2, z.B. dadurch, daß Gravitation selbst der Gravitation unterliegt und somit weniger Kraft ausüben kann, wenn die Masse nur genügend groß ist?
    Falls das sogar für kleinere Massen bereits der Fall sein sollte: Vielleicht können wir deshalb G auch nicht so genau messen?

    Wie würde die Rotverschiebung von Licht verlaufen, das im Zentrum eines SL startet?
    Ist die Dichte im SL überall gleich?
    Im Zentrum herrscht zumindest Schwerelosigkeit…

    Vielleicht kannst Du ja Licht ins Dunkel bringen?

  142. #142 SCHWAR_A
    28. April 2011

    @Niels:
    Da ist wohl was schiefgelaufen… bitte streiche den ersten ‘Versuch’. Danke.

  143. #143 Wolfgang Flamme
    28. April 2011

    @Niels, perk

    Mal eine Frage: Wie groß kann die gravitative Zeitdilatation denn überhaupt werden im Bereich _vor_ dem Ereignishorizont (R>R_s), wo ‘die Schwarzschild-Koordinaten für die genaue Beschreibung der Situation anscheinend noch eine geeignete Wahl sind’? Gibt es eine absolute Grenze oder heißt die Einschränkung ‘beliebig groß, aber nicht unendlich’?

    Naja, Ihr seht sicher, worauf ich hinauswill; also seid bitte zart.

  144. #144 Christian 2
    29. April 2011

    Wenn die mittlere Dichte abnimmt- Hat das dann einen Einfluss auf Massen, bzw. nimmt die Fähigkeit Materie in größeren Mengen aufzunehmen oder festzuhalten irgendwann ab? Es gibt ja schwarze Löcher mit Milliarden Sonnenmassen. Dann ist aber auch die Frage inwieweit die Gravitation noch ausreicht, um die Galaxie selbst zusammenzuhalten.

  145. #145 Bjoern
    29. April 2011

    @Christian2: Äh, kein Mensch hat jemals behauptet, das SL im Zentrum einer Galaxie würde die Galaxie zusammen halten!

    Und die Fähigkeit, Materie festzuhalten, hängt übrigens nicht von der mittleren Dichte ab, sondern (bei vorgegebenem Abstand zum Zentrum) nur von der gesamten Masse.

  146. #146 Christian 2
    29. April 2011

    Zum Teil hält das SL die Galaxie schon zusammen. Eben aufgrund der großen Massenansammlung dort. Mir ist klar, das dies nicht der einzigste Grund ist (Dunkle Materie).
    Nun nehmen wir an das schwarze Loch wäre entsprechend groß, und hätte nur noch eine geringe Dichte im Vergleich zum Volumen, was ja aufgrund der Größe und des zunehmenden Abstands der Sonnen zum Zentrum eine schwächere Anziehungskraft vermuten lässt. Der Einfluss der Gravitation ist ja auch begrenzt.

    Die Galaxie ist dann auch entsprechend größer, vermutlich sehr alt. Daher die Frage, ob da Zusammenhänge bestehen könnten.

  147. #147 Wolfgang Flamme
    11. Mai 2011

    Tja, so geht’s mir irgendwie immer bei dem Thema ‘Schwarzes Loch (trivial)’, seit nunmehr gut 30 Jahren: Ich stelle naheliegende Fragen und die Antworten der Wissenden bleiben irgendwann einfach aus. Dabei will ich gar nicht die ART infrage stellen – bin sogar zutiefst überzeugt, daß sie richtig ist – ich will einfach nur wissen, wie man angeblich schon jetzt die Auswirkungen von etwas beobachten können soll, das aufgrund der Zeitdilatation eigentlich erst im Entstehen begriffen sein sollte.

    Dabei könnte ich sogar damit umgehen, wenn man mir (mit Hinweis auf dieses und jenes) vorhalten würde, ich sei einfach nur vernagelt oder zu blöde. Da bliebe mir zumindest mal der erhoffte Hinweis, wo mein Denkfehler liegt. Aber so bleibt mir nichtmal das.

  148. #148 Florian Freistetter
    11. Mai 2011

    @Wolfgang Flamme: “Tja, so geht’s mir irgendwie immer bei dem Thema ‘Schwarzes Loch (trivial)’, seit nunmehr gut 30 Jahren: Ich stelle naheliegende Fragen und die Antworten der Wissenden bleiben irgendwann einfach aus.”

    Naja – alle hier beteiligten machen das nicht hauptberuflich. Sondern in ihrer Freizeit. Und manchmal fehlt die halt. Manchmal sagt man auch deswegen nichts, weil man kein Experte auf dem Gebiet ist. Manchmal ists auch einfach nur Pech.

  149. #149 Wolfgang Flamme
    11. Mai 2011

    Florian,
    es ist ja auch keine Anklage sondern nur eine Klage 🙂

  150. #150 Christian 2
    11. Mai 2011

    @ Florian: Du musst trotz deines Hobbyinteresses an schwarzen Löchern zugeben das dies Dinge sind bei denen auch Experten nur spekulieren können, wie ihre wahre Natur vielleicht beschaffen ist.

    Schwarze Löcher sind für mich eine Grenze der Physik, vergleichbar mit der einer gedachten Grenze am Ende des Universums. Oder anderen Grenzen, wie die Schwelle zum Tod.
    Wir kommen dort nur mit theoretischen Hilfskonstrukten weiter. Aber wir können niemals dort hin und nachprüfen, ob unsere Vermutungen auch stimmen.

    Schwarze Löcher liegen fernab von dem, was ein menschlicher Verstand sich ab einem gewissen Punkt noch sinnvoll vorstellen könnte. Die Zeitdilatation z.B. überschreitet unsere Vorstellungskraft.
    Darüber nachzugrübeln macht wie ich finde für Otto Normal wenig Sinn. Man wird einfach nie schlau draus, egal wie lange man darüber nachdenkt.

    Die Sache wird irgendwann zu abstrus, und man verzichtet gewöhnlich darauf noch einen Schritt weiter zu gehen. Unendlichkeiten sind eben schwer vorstellbar.

    Es ist mal ganz nett darüber zu lesen, das sich Wissenschaftler wie Hawking mit einer Naturbegabung für die Vorstellung mehrdimensionaler Räume ernsthaft Gedanken zum Thema machen.
    Ob es Sinn macht etwas theoretisch zu erforschen dessen Überprüfbarkeit nach unserem heutigen Wissensstand engen Grenzen gesetzt ist, muss sich noch zeigen.

  151. #151 Florian Freistetter
    12. Mai 2011

    @Christian: “Du musst trotz deines Hobbyinteresses an schwarzen Löchern zugeben das dies Dinge sind bei denen auch Experten nur spekulieren können, wie ihre wahre Natur vielleicht beschaffen ist. S”

    Und? Das nennt sich Wissenschaft. Hypothesen aufstellen. Spekulieren. Überprüfen. Verwerfen. usw.

    “Aber wir können niemals dort hin und nachprüfen, ob unsere Vermutungen auch stimmen.”

    Warum nicht?

    “Schwarze Löcher liegen fernab von dem, was ein menschlicher Verstand sich ab einem gewissen Punkt noch sinnvoll vorstellen könnte. “

    So wie z.B. die gesamte Quantenmechanik. Trotzdem ist diese Theorie wunderbar experimentell überprüfbar und überprüft.

    “Ob es Sinn macht etwas theoretisch zu erforschen dessen Überprüfbarkeit nach unserem heutigen Wissensstand engen Grenzen gesetzt ist, muss sich noch zeigen. “

    Nochmal: das ist Wissenschaft. Fändest du es sinnvoller wenn man nur das erforscht, was man eh schon kennt?

  152. #152 Bullet
    12. Mai 2011

    Und hier liegt der Hase im Pfeffer:

    Schwarze Löcher sind für mich eine Grenze der Physik

    Und für den Michl ist Klein-Kleckersdorf der Rand der Welt.

  153. #153 Christian 2
    13. Mai 2011

    @ Florian: Es geht darum, das man gewisse Grenzen nicht überschreiten kann. Der Schwarzschildradius bzw. alles, was dahinter liegt ist bleibt pure Theorie.
    Wie soll man etwas nachprüfen, was sich der Überprüfbarkeit aufgrund physikalischer Gesetzmäßigkeiten entzieht?

    Das der Mensch dazu neigt bei nicht überprüfbaren Theorien ordentlich in den Kufladen zu langen, brauche ich glaub ich nicht weiter zu vertiefen. Trotzdem werden sie erdacht und berechnet. Der eigentliche Sinn dahinter ist mir unklar.

  154. #154 Florian Freistetter
    13. Mai 2011

    @Christian 2: “Der eigentliche Sinn dahinter ist mir unklar. “

    Ja, DIR ist er unklar. Das heisst aber nicht, dass es sinnlos ist zu forschen.

    “Wie soll man etwas nachprüfen, was sich der Überprüfbarkeit aufgrund physikalischer Gesetzmäßigkeiten entzieht?”

    Abgesehen davon stimmt das ja auch nicht. Hawking-Strahlung, etc…

    Der Mensch kann nur sehr wenig seiner Umwelt direkt wahrnehmen. Deswegen haben wir uns ja auch indirekte Methoden ausgedacht.

  155. #155 Tim
    17. Januar 2012

    Hallo Florian,

    ich verfolge gerne und mit großer Begeisterung Deinen Blog (aufmerksam geworden durch den CRE Podcast mit Tim Pritlove “Sonnensystem”).
    Speziell das Thema der schwarzen Löcher finde ich extrem spannend.

    Mir stellt sich zur Zeit die Frage, wie sich das aktuelle Gedankenmodell ändert in Bezug der neuen “Erkenntnisse” aus den letzten CERN Beobachtungen was die Lichtgeschwindigkeit angeht.
    Ich bin nicht auf dem Stand ob sich die Beobachtungen nun verhärtet haben, dass Neutrinos (waren es glaub ich) schneller als Licht sind, auch wenn es nur ein Bruchteil ist. 😉

    Könnte das denn nun evtl. bedeuten, dass es also unter Umständen doch möglich ist ein schwarzes Loch zu verlassen? Ist das evtl. auch der Grund warum schwarze Löcher eben nicht einfach schwarz sind sondern Strahlung abgeben?

  156. #156 Florian Freistetter
    17. Januar 2012

    @Tim: “Ist das evtl. auch der Grund warum schwarze Löcher eben nicht einfach schwarz sind sondern Strahlung abgeben? “

    In Sachen CERN-Neutrinos gibts nix Neues; so schnell wird sich da auch nichts tun. Und das schwarze Löcher Strahlung abgeben, hat Hawking schon in den 1970er Jahren rausgefunden 😉 https://de.wikipedia.org/wiki/Hawking-Strahlung